Крок 2 - Медицина 2021 1 день (буклет)

1 / 150
В органiзм людини з атмосферного повiтря надходять декiлька хiмiчних речовин. Як називається тип комбiнованої дiї, при якому сумiсний ефект менший вiд суми ефектiв кожної з речовин, що входить у комбiнацiю, при їх iзольованiй дiї на органiзм? Several chemical substances enter the human body from the atmospheric air. What is the type of combined action called, in which the combined effect is less than the sum of the effects of each of the substances included in the combination , with their isolated effect on the organism?

Антагонізм Antagonism

Поєднана дія Combined Action

Ізольована дія Isolated Action

Комплексна дія Complex Action

Потенціювання Potentiation

2 / 150
Жінка хворіє на бронхіальну астму із супутнім цукровим діабетом І типу, отримує в комплексній терапії: сальметерол, беклометазон, еуфілин, кромолін та преднізолон. Після призначення якої з наведених груп препаратів потрібна корекція дози інсуліну? A woman suffers from bronchial asthma with concomitant type 1 diabetes mellitus, receives in complex therapy: salmeterol, beclomethasone, euphylline, cromolyn and prednisolone. After the appointment of which of the following groups do you need insulin dose correction?

Метил ксантин Methylxanthine

Інгаляційні глюкокортикоїди Inhaled glucocorticoids

Системні глюкокортикоїди Systemic glucocorticoids

Кромони Cromoni

Пролонговані b2-агоністи Prolonged b2-agonists

3 / 150
Працівник заводу з виробницва фарб скаржиться на неприємний смаку роті, переймоподібні болі в животі, закрепи, які не знімаються послаблюючими засобами. Об'єктивно встановлено: шкіра обличчя блідо-сіруватого кольору, по краю ясен біля передніх зубів - смужка сіровато-бузкового кольору. Під час лабораторного дослідження виявлено: у крові анемію, ретикулоцитоз, базофільну зернистість еритроцитів, педвищений вміст білірубіну; у сечі - підвищений вміст порфірину. Яке захворювання у робітника? An employee of a paint production plant complains of an unpleasant taste in the mouth, spasm-like pains in the stomach, constipation that cannot be removed by laxatives. Objectively established: the skin of the face is pale grayish in color, along the edge of the gums near the front teeth - a strip of grayish-lilac color. During the laboratory examination, the following were found: anemia, reticulocytosis, basophilic granularity of erythrocytes, an increased content of bilirubin in the blood; an increased content of porphyrin in the urine. What disease does the worker have?

Хронічне отруєння чотирихлористим вуглецем Chronic carbon tetrachloride poisoning

Хронічне отруєння ртуттю Chronic mercury poisoning

Хронічне отруєння бензолом Chronic benzene poisoning

Хронічне отруєння марганцем Chronic manganese poisoning

Хронічне отруєння свинцем Chronic lead poisoning

4 / 150
Чоловік 45 років звернувся до лікаря зі скаргами на дратівливість, підвищену втомленість, схуднення, серцебиття, перебої в роботі серця. Під час пальпації лівої долі щитоподібної залози пальпується утворення щільно-еластичної консистенції, безболісне, яке під час ковтання зміщується разом із щитоподібною залозою. Позитивні очні симптоми, екзофтальм, порушення конвергенції. Артеріальний тиск -135/80 мм рт. ст., пульс - 110/хв. Під час ультразвукового дослідження щитоподібної залози у лівій долі візуалізується гіперехогенне округле утворення розмірами 2x3 см, з чіткими контурами, однорідної структури. Який висновок лікаря після проведеного обстеження буде найбільш коректним? A 45-year-old man turned to the doctor with complaints of irritability, increased fatigue, weight loss, palpitations, heart failure. During palpation of the left lobe of the thyroid gland, a mass is palpated densely - elastic consistency, painless, which during swallowing moves together with the thyroid gland. Positive eye symptoms, exophthalmos, convergence disorder. Blood pressure - 135/80 mm Hg, pulse - 110/min. During ultrasound examination of the thyroid gland in in the left lobe, a hyperechoic rounded formation measuring 2x3 cm, with clear contours, of a uniform structure is visualized. What conclusion of the doctor after the examination will be the most correct?

Рак щитоподібної залози Thyroid cancer

Гострий тиреоїдит Acute thyroiditis

Вузол щитоподібної залози Thyroid nodule

Кіста щитоподібної залози Thyroid cyst

Дифузно-токсичний зоб Diffuse toxic goiter

5 / 150
Чоловік скаржиться на гнійні виділення з пенісу вранці перед сечовиділенням, різі, полакіурію. Визначена піурія під час проведення трьохстаканної проби в першій порції сечі. Який імовірний діагноз? A man complains of purulent discharge from the penis in the morning before urinating, cuts, pollakiuria. Pyuria was detected during a three-glass test in the first portion of urine. What is the probable diagnosis?

Простатит Prostatitis

Цистит Cystitis

Пієлонефрит Pyelonephritis

Баланопостит Balanoposthitis

Уретрит Уретрит

6 / 150
Мати хлоптичка 3 років під час купання дитини виявила пухлиноподібний утвір у животі. При обстеженні визначається щільний, не болючий, з рівною поверхнею, малорухомий пухлиноподібний утвір великих розмірів, розташований під лівою реберною дугою. Температура тіла 37,5о С, ЧСС-110 на 1 хв., АТ -150/90 мм.рт.ст. У загальному аналізі крові Er 3,2х1012 , Hb 90 г/л, Le 10,4х109 л, ШОЕ 35 мм/год. У загальному аналізі сечі – еритроцити на все поле зору. Яке захворювання можна запідозрити у дитини? The mother of a 3-year-old boy discovered a tumor-like mass in the abdomen while bathing the child. During the examination, a dense, non-painful, flat-surfaced, slow-moving tumor-like mass of large sizes, located under the left costal arch. Body temperature 37.5oC, heart rate-110 for 1 minute, blood pressure -150/90 mmHg. In the general blood test, Er 3.2x1012, Hb 90 g/l, Le 10.4x109 l, ESR 35 mm/h. In the general analysis of urine, erythrocytes are visible throughout the field of vision. What disease can be suspected in a child?

Карбункул нирки Kidney carbuncle

Крижово-куприкова тератома Sacral-coccygeal teratoma

Гострий пієлонефрит Acute pyelonephritis

Вроджений гідронефроз Congenital hydronephrosis

Пухлина Вільямса Williams Tumor

7 / 150
Хлопчик 8-ми років, звернувся до ліка­ря зі скаргами на підвищену втомлюваність, зниження апетиту, іктеричність склер, біль у животі. У періоді новонародженості була затяжна жовтяниця. У фізичному розвитку не відстає. Об'єктивно: шкіра бліда, сли­зові оболонки і склери іктеричні. Печінка +2 см, злегка болюча при пальпації. Ви­порожнення і сечовиділення в нормі. При лабораторному дослідженні крові: еритро­цити - 4,5 • 1012/л. гемоглобін - 115 г/л, лей­коцити - 7 • 109/л, швидкість осідання ери­троцитів - 8 мм/год, білірубін прямий - 10 мкмоль/л. непрямий - 39 мкмоль/л, аспартатамінотрансфераза (АСТ) - 17 Од/л, аланінамінотрансфераза (АЛТ) - 21 Од/л. Який діагноз є найбільш імовірним? An 8-year-old boy turned to the doctor with complaints of increased fatigue, decreased appetite, icterus of the sclera, abdominal pain. In the newborn period, he had prolonged jaundice. In does not lag behind in physical development. Objectively: the skin is pale, mucous membranes and sclera are icteric. Liver +2 cm, slightly painful on palpation. Defecation and urination are normal. In the laboratory blood test: erythrocytes - 4.5 • 1012/l. hemoglobin - 115 g/l, leukocytes - 7 • 109/l, erythrocyte sedimentation rate - 8 mm/h, direct bilirubin - 10 μmol/l, indirect - 39 μmol/l, aspartate aminotransferase (AST) - 17 U/l, alanine aminotransferase ( ALT) - 21 units/l. What diagnosis is the most probable?

Гемолітична анемія Hemolytic anemia

Дискінезія жовчовивідних шляхів Biliary tract dyskinesia

Синдром Жильбера Gilbert's Syndrome

Хронічний вірусний гепатит Chronic viral hepatitis

8 / 150
Жінка 37 років, хімік-технолог. Турбує поява синців на животі, стегнах, гомілках після невеликих травм або без причини. Під час огляду виявлені численні дрібні та дещо більші синці, які місцями зливаються (від 1х1 до 4х5 см). Інших змін немає. Менструація триває до 7 днів. Селезінка не пальпується. В крові: НЬ- 92 г/л, лейкоцити - 7, 2 • 109/л, тромбоцити - 6 • 109/л, швидкість осідання еритроцитів - 33 мм/год. Який патогенез даного захворювання? A 37-year-old woman, a chemist-technologist. She is concerned about the appearance of bruises on the abdomen, thighs, lower legs after minor injuries or for no reason. During the examination, numerous small and slightly larger ones were found bruises that merge in places (from 1x1 to 4x5 cm). There are no other changes. Menstruation lasts up to 7 days. The spleen is not palpable. In the blood: Hb - 92 g/l, leukocytes - 7, 2 • 109/l, platelets - 6 • 109/l, erythrocyte sedimentation rate - 33 mm/h. What is the pathogenesis of this disease?

Підвищення фібринолітичної активності Increased fibrinolytic activity

Зниження рівня протромбіну крові Decreased blood prothrombin level

Скорочення тривалості життя тромбоцитів Decreased life expectancy of platelets

Зниження рівня антигемофільного глобуліну Decrease in antihemophilic globulin

Зниження рівня фібриногену Decrease in fibrinogen

9 / 150
Хвора 38 років звернулася до жіночої консультації зі скаргами на помірні кров'янисті виділення зі статевих шляхів, які виникли після затримки чергової менструації на 1,5 місяці. Під час вагінального обстеження виявлено: шийка матки неерозована, симптом «зіниці» (+++); матка не збільшена, щільна, рухома, неболюча; придатки з обох боків незбільшені, неболючі; склепіння глибокі. Який діагноз найімовірніший? A 38-year-old patient turned to a women's consultation with complaints of moderate bleeding from the genital tract, which occurred after the next menstruation was delayed for 1.5 months. During vaginal examination revealed: the cervix is not eroded, the 'pupil' symptom (+++); the uterus is not enlarged, dense, mobile, painless; the appendages on both sides are not enlarged, painless; the vaults are deep. What is the most likely diagnosis?

Маткова вагітність Uterine pregnancy

Позаматкова вагітність Ectopic pregnancy

Внутрішній ендометріоз тіла матки Internal endometriosis of the uterine body

Дисфункціональна маткова кровотеча Dysfunctional uterine bleeding

Рак тіла матки Cancer of the uterine body

10 / 150
У чоловіка 52 років, який звернувся до гастроентеролога з приводу виразки шлунка, виявлено диспластичне ожиріння, стриї на животі та стегнах. Артеріальний тиск -170/100 ми рт. ст. Під час лабораторного обстеження виявлено підвищений рівень кортизолу, помірно знижений рівень АКТГ. Який імовірний діагноз? A 52-year-old man, who consulted a gastroenterologist for a stomach ulcer, was found to have dysplastic obesity, striae on the abdomen and thighs. Blood pressure -170/100 mmHg. Art. During the laboratory examination, an elevated cortisol level and a moderately reduced ACTH level were detected. What is the probable diagnosis?

Аліментарне ожиріння Alimentary obesity

Гіпоталамічний синдром з ендокринно-обмінними порушеннями Hypothalamic syndrome with endocrine-metabolic disorders

Синдром Іценка-Кушинга Itsenko-Cushing syndrome

Хвороба Іценка-Кушинга Itsenko-Cushing disease

Синдром Золінгера-Еллісона Zollinger-Ellison Syndrome

11 / 150
На підприємстві з виробництва синтетичних миючих речовин збільшилися випадки захворювання ринітами, вазоринітами, фарингітами, дерматитами. У повітрі робочої зони присутній пил миючих речовин. Фахівці вважають, що клінічні прояви й зростання захворюваності зумовлене впливом пилу. Які особливості дії пилу полягають в основі патогенезу? At the enterprise for the production of synthetic detergents, cases of rhinitis, vasorinitis, pharyngitis, and dermatitis have increased. The air of the working area contains dust from detergents. Experts believe that the clinical manifestations and the increase in morbidity is due to the impact of dust. What features of the action of dust are the basis of pathogenesis?

Загальнотоксична дія General toxic effect

Подразнююча дія Annoying action

Фіброгенна дія Fibrogenic effect

Канцерогенна дія Carcinogenic action

Алергічна дія Allergic action

12 / 150
Чоловік 56 років хворіє на бронхоектатичну хворобу впродовж 18 років. Протягом останніх півроку спостерігаються виражені набряки гомілок, обличчя. У протеїнограмі зазначено: загальний білок - 52 г/л, альбумін-глобуліновий коефіцієнт - 0,7, а2-глобуліни -14%, β-глобуліни -17%, у-глобуліни — 24 г/л. ШОЕ - 52 мм/год. Добова протеїнурія -4,5 г. ЕКГ: синусова тахікардія. Яке захворювання є найбільш імовірне? A 56-year-old man has been suffering from bronchiectasis for 18 years. Over the past six months, severe swelling of the legs and face has been observed. The proteinogram shows: total protein - 52 g/l, albumin-globulin ratio - 0.7, a2-globulins -14%, β-globulins -17%, y-globulins - 24 g/l ESR - 52 mm/h Daily proteinuria -4.5 g ECG: sinus tachycardia. What disease is most likely?

Хронічне легеневе серце Chronic pulmonary heart

Амілоїдоз нирок Kidney amyloidosis

Туберкульоз нирок Kidney tuberculosis

Хронічний гломерулонефрит Chronic glomerulonephritis

Хронічний пієлонефрит Chronic pyelonephritis

13 / 150
У хлопця 18 років скарги на переймоподібний біль в животі, рідкі випорожнення до 6 разів/добу з домішками слизу та свіжої крові. Хворіє впродовж року. Схуд на 10 кг. Об-но: Живіт м’який, болючий по ходу товстої кишки, особливо зліва. Сигмовидна кишка спазмована. В крові: ер. – 3,2х1012/л, Нв – 92 г/л, лейк. – 10,6х109/л, ШЗЕ – 34 мм/год. Іригоскопія – товста кишка звужена, гаустри відсутні, контури нечіткі, симптом “водопровідної труби”. Який найбільш ймовірний діагноз? An 18-year-old boy complains of spasm-like abdominal pain, loose stools up to 6 times/day with impurities of mucus and fresh blood. He has been ill for a year. He lost 10 kg General information: The abdomen is soft, painful along the course of the colon, especially on the left side. The sigmoid colon is spasmodic. In the blood: er - 3.2x1012/l, Hb - 92 g/l, leuk - 10.6x109/l , SZE - 34 mm/h. Irigoscopy - the colon is narrowed, the haustris are absent, the contours are unclear, the 'water pipe' symptom. What is the most likely diagnosis?

Туберкульоз кишечника Intestinal tuberculosis

Хронічний ентероколіт Chronic enterocolitis

Амебна дизентерія Amoebic dysentery

Неспецифічний виразковий коліт Nonspecific ulcerative colitis

Хвороба Крона Crohn's disease

14 / 150
Чоловік підібраний на вулиці в непритомному стані та доставлений до санпропускника. У нього спостерігаються повторні напади тонічних і клонічних судом рук і ніг з невеликими проміжками, мимовільне сечовипускання. Зіниці широкі, не реагують на світло. На язиці -сліди прикушування. Вогнищева неврологічна симптоматика не виявлена. AT -140/90 мм рт. ст. Голова хворого може бути вільно приведена до грудей. Визначіть найбільш імовірну патологію: A man was picked up on the street in an unconscious state and taken to the medical examiner. He has repeated attacks of tonic and clonic convulsions of the arms and legs with short intervals, involuntary urination. Pupils are wide , do not react to light. There are bite marks on the tongue. Focal neurological symptoms are not detected. AT -140/90 mm Hg. The patient's head can be freely brought to the chest. Determine the most likely pathology:

Інфаркт мозку Cerebral infarction

Епілептичний статус Status epilepticus

Гострий менінгоенцефаліт Acute meningoencephalitis

Правець Tetanus

Паренхіматозний крововилив Parenchymal hemorrhage

15 / 150
До дерматолога звернулася мати з дівчинкою 6 років. Дитина хворіє 5 днів. У дитини на тлі нежиті з'явилася велика кількість гнійної висипки. Об'єктивно встановлено: на шкірі обличчя є численні фліктени, наповнені серозно-гнійним вмістом, на деяких сформовані золотисто-жовті кірки. Який імовірний діагноз? A mother with a 6-year-old girl consulted a dermatologist. The child has been sick for 5 days. The child has a large amount of purulent rash against the background of a runny nose. Objectively established: on the skin of the face has numerous phlycten filled with serous-purulent contents, some have formed golden-yellow crusts. What is the probable diagnosis?

Токсико-алергічний дерматит Toxic-allergic dermatitis

Стрептококове імпетиго Streptococcal impetigo

Фотодерматит Photodermatitis

Атопічний дерматит Atopic dermatitis

Дитяча екзема Children's eczema

16 / 150
Жінка 45 років звернулась до лікарні за 12 діб після початку захворювання зі скаргами на біль та припухлість І пальця лівої кисті. Під час обстеження виявлено підвищення температури тіла до 38,9 °С. Нігтьова фаланга І пальця колбоподібно потовщена, синьо-багряного кольору. Під нігтьовою пластинкою та місцями під епідермісом видно гній. Під час пальпації відчуває різкий біль. На Ro-грамі пальця видні деструктивні зміни в кістці нігтьової фаланги. Який вид панарицію у хворої? A 45-year-old woman went to the hospital 12 days after the onset of the disease with complaints of pain and swelling of the I finger of the left hand. During the examination, an increase in body temperature up to 38, 9 °C. The nail phalanx of the I finger is bulb-shaped thickened, blue-purple in color. Pus is visible under the nail plate and in places under the epidermis. During palpation, a sharp pain is felt. On the Ro-gram of the finger, destructive changes in the bone of the nail phalanx are visible. What type of panaritium in sick?

Підшкіряний Subcutaneous

Суглобовий Articulated

Кістковий Bone

Сухожильний Tendon

Шкіряний Leather

17 / 150
Чоловік 35 років упродовж 14 років працює в ливарному цеху, де концентрація кварцевого пилу в 4 рази перевищує ГДК. Упродовж 4 років його турбує кашель, задишка під час фізичного навантаження. Під час аускультації виявлено ослаблене дихання. Рентгенографія ОГК встановила: емфізема, дрібноплямисті тіні по всіх легеневих полях. Який найбільш імовірний діагноз? A 35-year-old man has been working in a foundry for 14 years, where the concentration of quartz dust is 4 times higher than the MPC. For 4 years, he has been troubled by cough, shortness of breath during physical exertion . During auscultation, weak breathing was detected. X-ray of OGK established: emphysema, fine-shaped shadows in all lung fields. What is the most likely diagnosis?

Хронічний бронхіт Chronic bronchitis

Туберкульоз легенів Pulmonary tuberculosis

Силікоз Silicosis

ХОЗЛ ХОЗЛ

Азбестоз Asbestosis

18 / 150
У породіллі 29 років на 3 добу після операції кесарського розтину з'явилась блювота, сильний біль внизу живота, потім по всьому животу, затримка стула та газів. Загальний стан важкий. Шкіряні покрови бліді, губи та язик сухі. Температура тіла - 39,4 °С. Живіт вздутий, симптоми подразнення очеревини позитивні у всіх відділах. Перистальтика кишківника послаблена. Діурез знижений. Матка велика, болюча під час пальпації, з неприємним запахом виділень із піхви. Про яку патологію можна думати? On the 3rd day after cesarean section, a 29-year-old woman in labor developed vomiting, severe pain in the lower abdomen, then throughout the abdomen, stool and gas retention. General condition severe. The skin is pale, the lips and tongue are dry. The body temperature is 39.4 °C. The abdomen is distended, symptoms of peritoneal irritation are positive in all departments. Intestinal peristalsis is weakened. Diuresis is reduced. The uterus is large, painful during palpation, with foul-smelling secretions from the vagina. What pathology can you think about?

Інфекція післяопераційної рани Postoperative wound infection

Пельвіоперитоніт Pelvioperitonitis

Панметрит Panmetrite

Метроендометрит Metroendometritis

Розповсюджений перитоніт Disseminated peritonitis

19 / 150
Чоловік 42 років госпіталізований до клініки з діагнозом: бронхоектатична хвороба, загострення. На рентген знімках бронхоектази локалізовані в межах окремих сегментів однієї долі. Яка подальша тактика лікування? A 42-year-old man was hospitalized with a diagnosis of bronchiectasis, exacerbation. On x-rays, bronchiectasis are localized within separate segments of one lobe. What is the further treatment strategy?

Консервативне лікування Conservative treatment

Антибактеріальна терапія Antibacterial therapy

Хірургічне лікування Surgical treatment

Дихальна гімнастика Breathing gymnastics

Диспансерне спостереження Dispensary observation

20 / 150
32-річна вагітна у терміні 5-6 тижнів була вакцинована проти грипу неживою вакциною разом з усією родиною. На той момент про вагітність вона не знала. Вагітність бажана. Пацієнтка звернулась до сімейного лікаря для отримання консультації щодо можливого впливу вакцини на розвиток і перебіг вагітності, виникнення вад розвитку у плода. Яку пораду слід надати вагітній? A 32-year-old pregnant woman was vaccinated against influenza with a non-live vaccine in the period of 5-6 weeks, together with the whole family. At that time, she did not know about the pregnancy. Pregnancy is desirable. The patient turned to the family doctor for advice on the possible impact of the vaccine on the development and course of pregnancy, the occurrence of developmental defects in the fetus. What advice should be given to the pregnant woman?

УЗД для виявлення вад розвитку плода Ultrasound to detect fetal malformations

Тест на антитіла до вірусу грипу Test for antibodies to influenza virus

Обов'язкова консультація інфекціоніста і генетика Mandatory consultation of an infectious disease specialist and geneticist

Запропонувати медикаментозний аборт Offer medical abortion

Вакцинація проти грипу є безпечною протягом вагітності Flu vaccination is safe during pregnancy

21 / 150
Чоловік 50 років скаржиться на наявність висипу на шкірі тулуба без суб'єктивних відчуттів, хворіє 3 місяці, не лікувався, висип прогресує. Об'єктивно встановлено: на шкірі тулуба дисемінована висипка у вигляді червоних негострозапальних лентикулярних та нумулярних папул із чіткими границями, укритих сріблястими лусочками, які легко знімаються при пошкрябуванні предметним склом, інтенсивніше пошкрябування призводить до появи крапкової кровотечі. Який імовірний діагноз? A 50-year-old man complains of the presence of a rash on the skin of the trunk without subjective sensations, he has been ill for 3 months, was not treated, the rash is progressing. Objectively established: on the skin trunk, a disseminated rash in the form of red non-acute inflammatory lenticular and nummular papules with clear borders, covered with silver scales, which are easily removed when scraped with a glass slide, more intensive scraping leads to the appearance of spot bleeding. What is the probable diagnosis?

Червоний плоский лишай Red lichen planus

Рожевий пітиріаз Pityriasis rosea

Екзема Eczema

Псоріаз звичайний Psoriasis usual

Дерматит Dermatitis

22 / 150
Дівчина 16 років прийшла на профілактичний медичний огляд. Під час збору анамнезу вона повідомила, що у 3 роки перехворіла на вітряну віспу. Інших серйозних захворювань в анамнезі немає. Батьки відмов від щеплень не писали, вакцинувалася у дитячому садочку та школі відповідно до Національного календаря профілактичних щеплень. Фізикальне обстеження без відхилень. Призначення яких вакцин буде найбільш доречним зараз? A 16-year-old girl came for a preventive medical examination. During the anamnesis, she reported that she had chickenpox at the age of 3. There are no other serious diseases in the anamnesis. Parents vaccinations were not refused, she was vaccinated in kindergarten and school in accordance with the National Calendar of Preventive Vaccinations. Physical examination without abnormalities. Which vaccines would be the most appropriate to prescribe now?

АДП-М АДП-М

Гепатит В Hepatitis B

Кпк Кпк

Гепатит А Hepatitis A

АаКДП АаКДП

23 / 150
Чоловік 47 років скаржиться на безсоння, важкість в усьому тілі, постійно пригнічений настрій. Вважає себе нікчемним, неспроможним. Гадає, що є тягарем для своїх рідних, воліє померти. Пригнічений, малорухливий, обличчя гіпомімічне, скорботне. Мова тиха, монотонна, відповіді короткі. Який діагноз є найбільш імовірним? A 47-year-old man complains of insomnia, heaviness in the whole body, a constantly depressed mood. He considers himself worthless, incapable. He thinks that he is a burden to his relatives, he prefers to die . Depressed, sedentary, face hypomimic, sorrowful. Speech is quiet, monotonous, answers are short. What diagnosis is most likely?

Великий депресивний розлад Major Depressive Disorder

Шизофренія з пізнім дебютом Schizophrenia with late onset

Невротична депресія Neurotic depression

Атеросклеротична депресія Atherosclerotic depression

Початкова стадія хвороби Альцгеймера Initial stage of Alzheimer's disease

24 / 150
Під час проходження допризивної комісії у хлопця 17 років виявлена артеріальна гіпертензія - AT на руках 190/110 мм рт. ст. Скарг немає. Звертає на себе увагу непропорційна будова тіла - добре розвинений плечовий поясі недорозвинені нижні кінцівки. Який метод дослідження, найвірогідніше, найбільш інформативний для постановки діагнозу? During the pre-draft commission, a 17-year-old boy was diagnosed with arterial hypertension - AT on the hands 190/110 mm Hg. No complaints. Disproportionate structure attracts attention bodies - a well-developed shoulder girdle, underdeveloped lower limbs. What research method is most likely to be the most informative for making a diagnosis?

УЗД нирок і наднирників Ultrasound of kidneys and adrenal glands

Реносцинтіграфія Renoscintigraphy

Вимірювання AT на нижніх кінцівках AT measurements on lower limbs

Доплер-сонографія судин Doppler sonography of vessels

Визначення катехоламінів у сечі Determination of catecholamines in urine

25 / 150
У дитини 10 місяців на тлі легкого перебігу ГРВІ з'явилися повторні клонічні судоми. ПІД час огляду констатовано чіткі прояви рахіту середньої важкості. Рівень кальцію крові -1,6 ммоль/л, інтервал Q-T на ЕКГ подовжений. Даних про перинатальне ушкодження ЦНС немає. Спинно-мозкова рідина інтактна, витікала підтиском. Вигодовування штучне, без овочевих страв. Яке захворювання проявилося на тлі ГРВІ? A 10-month-old child developed repeated clonic convulsions against the background of a mild acute respiratory viral infection. DURING the examination, clear manifestations of moderate rickets were noted. Blood calcium level -1.6 mmol/l, the QT interval on the ECG is prolonged. There are no data on perinatal damage to the central nervous system. The cerebrospinal fluid is intact, it leaked out under negative pressure. Feeding is artificial, without vegetable dishes. What disease manifested itself against the background of ARVI?

Енцефаліт Encephalitis

Менінгіт Meningitis

Нейротоксикоз Neurotoxicosis

Енцефалітична реакція Encephalitic reaction

Спазмофілія Spasmophilia

26 / 150
Чоловік 60 років скаржиться на часті болісні утруднені сечовипускання, переривчастий струмінь сечі. Уночі сечовипускання до 5 разів. Після фізичного навантаження відзначає домішки крові у сечі. Ректально встановлено: простата збільшена, щільно-еластична, безболісна, без вузлів. У сечі спостерігається; питома вага -1020, білок - 0,1 г/л, лейкоцити -20-30 у п/з, еритроцити - 10-20 у п/з. Який метод обстеження першочерговий? A 60-year-old man complains of frequent, painful, difficult urination, an intermittent stream of urine. At night, he urinates up to 5 times. After physical exertion, he notes blood impurities in the urine. Rectal diagnosis: prostate enlarged, dense-elastic, painless, without knots. It is observed in urine: specific gravity -1020, protein - 0.1 g/l, leukocytes -20-30 in p/z, erythrocytes - 10-20 in p/z. What is the examination method primary?

Бактеріологічне дослідження сечі Urine bacteriological examination

Оглядова урографія Review urography

Екскреторна урографія Excretory urography

Цистоскопія Cystoscopy

Трансректальне ультразвукове дослідження Transrectal ultrasound examination

27 / 150
У малюка 9 місяців спостерігається затримання розвитку зубів та подовжується строк зарощення тім'ячка, слабкість та пітнівість. Який вид гіповітамінозу може бути у малюка? A 9-month-old baby has a delay in the development of teeth and a prolonged period of hair growth, weakness and sweating. What kind of hypovitaminosis can a baby have?

Гіповітаміноз А Hypovitaminosis A

Гіповітаміноз D Hypovitaminosis D

Гіповітаміноз В1 Hypovitaminosis B1

Гіповітаміноз С Hypovitaminosis C

Гіповітаміноз В6 Hypovitaminosis B6

28 / 150
Чоловік 45 років має скарги на постійний головний біль, біль у серці, колінних суглобах. Три роки тому був укус кліща з кільцевою еритемою на шкірі. Об'єктивно встановлено: акродерматит, двобічна пірамідна недостатність, розлади координації. Який діагноз найбільш імовірний? A 45-year-old man complains of constant headache, pain in the heart, and knee joints. Three years ago he was bitten by a tick with annular erythema on the skin. Objectively established : acrodermatitis, bilateral pyramidal insufficiency, coordination disorders. What is the most likely diagnosis?

Нейробореліоз Neuroborreliosis

Вірусний енцефаліт Viral encephalitis

Нейросифіліс Neurosyphilis

Розсіяний склероз Multiple sclerosis

Ішемічна енцефалопатія Ischemic encephalopathy

29 / 150
Працівник 39 років звернувся до лікаря зі скаргами на ниючии біль та відчуття оніміння у кистях та передпліччях, зниження м'язової сили рук, порушення сну, роздратованість та зниження слуху. Дані професійного анамнезу свідчать про те, що робітник протягом 12 років працює бурильником, використовуючи свердло вагою 20 кг. Об'єктивно встановлено: шкіра кистей має синюшний відтінок, відмічається набряк кінчиків пальців, стертість шкірного малюнка, легка деформація міжфалангових суглобів, зниження тактильної, температурної та больової чутливості. Який найбільш імовірний діагноз? A 39-year-old employee went to the doctor with complaints of pain and numbness in the hands and forearms, decreased arm muscle strength, sleep disturbances, irritability, and hearing loss . Data from the professional history indicate that the worker has been working as a driller for 12 years, using a drill weighing 20 kg. Objectively established: the skin of the hands has a bluish tint, edema of the fingertips, roughness of the skin pattern, slight deformation of the interphalangeal joints, decreased tactile, temperature and pain sensitivity. What is the most likely diagnosis?

Нервово-циркуляторна дистонія Nervous-circulatory dystonia

Вібраційна хвороба, зумовлена впливом локальної вібрації Vibration disease caused by exposure to local vibration

Хвороба Рейно Raynaud's disease

Ревматичний поліартрит Rheumatoid polyarthritis

Вібраційна хвороба, зумовлена впливом загальної вібрації Vibration disease caused by exposure to general vibration

30 / 150
Жiнка 52-х рокiв звернулася до лiкаря зi скаргами на швидку втомлюванiсть та задишку протягом останнього року з перiодичним сухим кашлем. Пацiєнтка вiдзначає, що останнiм часом їй стало важко пiднiмати обидвi руки та розчiсувати волосся. При фiзикальному обстеженнi температура тiла - 37,4 C, пульс - 76/хв., артерiальний тиск - 130/85 мм рт.ст., SpO2 - 95% при кiмнатному повiтрi. При неврологiчному дослiдженнi незначне зниження сили дельтоподiбного м’яза. При аускультацiї легень - розповсюдженi сухi хрипи. На щоках, навколо орбiт та на лiктях еритематозний висип. Який метод найбiльш iмовiрно пiдтвердить дiагноз у цiєї пацiєнтки? A 52-year-old woman came to the doctor with complaints of rapid fatigue and shortness of breath for the past year with periodic dry cough. The patient notes that recently it has become difficult for her to lift both hands and comb hair. On physical examination, body temperature - 37.4 C, pulse - 76/min, blood pressure - 130/85 mm Hg, SpO2 - 95% on room air. On neurological examination, a slight decrease in strength deltoid muscle. On auscultation of the lungs, there are widespread dry rales. On the cheeks, around the orbits, and on the elbows, an erythematous rash. What method is most likely to confirm the diagnosis in this patient?

Біопсія м'яза Muscle biopsy

Виявлення антинуклеарних антитіл Detection of antinuclear antibodies

Рентгенографія ОГП Roentgenography of OHP

Біопсія лімфовузла Lymph node biopsy

КТ легень CT lung

31 / 150
Дитина народилася від 1-ї вагітності, що протікала з гестозом 1 і 2-ї половини. Пологи відбулися шляхом кесаревого розтину. Оцінка за шкалою Апгар - 6 балів. Вага дитини - 3000 г. Груди смоктала активно, не відригувала. За два тижні з'явилося блювання «фонтаном» із домішками створоженого молока, об'єм перевищував з'їденДитина стала в'ялою, погано набирає вагу, зменшилася кількість сечі та випорожнень. Який імовірний діагноз? The child was born from the 1st pregnancy, which proceeded with gestosis of the 1st and 2nd half. The delivery took place by caesarean section. Apgar score - 6 points. The child's weight is 3000 g. She actively sucked her breasts, did not regurgitate. In two weeks, 'fountain' vomiting with impurities of colostrum appeared, the volume exceeded what she had eaten. What is the probable diagnosis?

Гостра кишечна інфекція Acute intestinal infection

Пілоростеноз Pylorostenosis

Пілороспазм Pilorospasm

Кишечна непрохідність Intestinal obstruction

Адреногенітальний синдром Adrenogenital syndrome

32 / 150
Чоловік 61 року звернувся до дільничного лікаря зі скаргами на напади стиснення за грудиною, які виникають під час ходьби до 200 м та зникають, якщо зупинитися. Уважає себе хворим близько року. Межі серця в нормі, тони помірно приглушені, 4CC=Ps=76/хв., AT -130/80 мм рт. ст. Який найбільш імовірний діагноз у хворого? A 61-year-old man turned to the district doctor with complaints of chest compression attacks that occur while walking up to 200 m and disappear when he stops. He considers himself ill year. Heart boundaries are normal, tones are moderately muffled, 4CC=Ps=76/min., AT -130/80 mm Hg. What is the most likely diagnosis for the patient?

ІХС. Нестабільна стенокардія IHD. Unstable angina

Тривожний розлад Anxiety disorder

ІХС. Стабільна стенокардія CHD. Stable angina

ІХС. Інфаркт міокарда без елевації сегменту ST CHD. Myocardial infarction without ST segment elevation

Хронічне обструктивне захворювання легень Chronic obstructive pulmonary disease

33 / 150
У породіллі вагою 70 кг. за 40 хвилин після пологів великим плодом (4500 г) з'явилися значні кров'янисті виділення зі статевих шляхів. Проведено випорожнення сечового міхура та зовнішній масаж матки; після чого матка скоротилася, але за 5 хвилин кровотеча відновилася. Після введення утеротопіків, ручного обстеження порожнини матки крововтрата склала 1200 мл. Якою повинна бути подальша тактика? In a woman in labor weighing 70 kg, 40 minutes after giving birth with a large fetus (4500 g), significant bloody discharge appeared from the genital tract. The bladder was emptied and external massage of the uterus; after that, the uterus shrank, but after 5 minutes the bleeding resumed. After the introduction of uterotopics, manual examination of the uterine cavity, the blood loss amounted to 1200 ml. What should be the further tactics?

Лапаротомія. Екстирпація матки без додатків Laparotomy. Extirpation of the uterus without attachments

Локальна гіпотермія Local hypothermia

Кюретаж стінок порожнини матки Curettage of the walls of the uterine cavity

Продовжити введення утеротоніків Continue administration of uterotonics

Компресія черевного відділу аорти Compression of the abdominal aorta

34 / 150
Дівчина 22 років скаржиться на дратівливість, плаксивість, головний біль, нудоту, іноді блювоту, серцебиття, нагрубання молочних залоз, набряки на руках і ногах, метеоризм. Скарги виникають за 6-10 днів до менструації та зникають із початком місячних. Бімануально встановлено: матка та додатки без змін. Який імовірний діагноз? A 22-year-old girl complains of irritability, tearfulness, headache, nausea, sometimes vomiting, palpitations, swelling of the mammary glands, swelling of the arms and legs, flatulence. Complaints occur 6-10 days before menstruation and disappear with the onset of menstruation. Bimanually established: the uterus and appendages are unchanged. What is the probable diagnosis?

Синдром Шихана Sheehan Syndrome

Передменструальний синдром Premenstrual syndrome

Нейро-циркуляторна дистонія Neuro-circulatory dystonia

Дисменорея Dysmenorrhea

Невроз Neurosis

35 / 150
Чоловік 45 років госпіталізований зі скаргами на біль, що виник раптово у лівій половині грудної клітки та епігастральній ділянці, утруднене дихання, задишку, нудоту, одноразову блювоту. Захворів гостро, після підняття великої ваги. Об'єктивно встановлено: дихання поверхневе, ЧДР - 38/хв, ліва половина грудної клітки відстає під час дихання. Під час перкусії прослуховується тимпанічний звук, дихання не прослуховується. Пульс - 110/хв, слабкого наповнення. AT - 100/60 мм рт. ст. Яка патологія розвинулась у хворого? A 45-year-old man was hospitalized with complaints of pain that appeared suddenly in the left half of the chest and epigastric area, difficulty breathing, shortness of breath, nausea, one-time vomiting. He became acutely ill , after lifting a heavy weight. Objectively established: breathing is superficial, CHDR - 38/min, the left half of the chest lags behind during breathing. During percussion, a tympanic sound is heard, breathing is not heard. Pulse - 110/min, weak filling. AT - 100/60 mm Hg. What pathology has developed in the patient?

Відкритий пневмоторакс Open pneumothorax

Закритий пневмоторакс Closed pneumothorax

Госта тампонада серця Host cardiac tamponade

Відрив хорди морального клапана Tear of the moral valve chord

Клапанний пневмоторакс Valvular pneumothorax

36 / 150
До лікаря звернулася жінка 45 років зі скаргами на загальну слабкість, підвищену втомлюваність, роздратованість, занепокоєння, зниження пам'яті, багаторазовий проніс, лущення та пігментацію відкритих ділянок тіла (шиї, кистей та підошв). Недостатність якого вітаміну може бути причиною такого стану пацієнта? A 45-year-old woman came to the doctor with complaints of general weakness, increased fatigue, irritability, anxiety, memory loss, frequent runny nose, peeling and pigmentation of open areas of the body (neck, hands and soles). Insufficiency of which vitamin can be the cause of this condition of the patient?

Рибофлавіну Riboflavin

Нікотинової кислоти Nicotinic acid

Фолієвої кислоти Folic acid

Тіаміну Thiamine

Ретинолу Retinol

37 / 150
У 3-місячної дитини на фоні субфебрильної температури тіла і риніту спостерігається блідість, ціаноз носогубного трикутника, виражена задишка експіраторного характеру, здута грудна клітка, сухий кашель, участь допоміжної мускулатури вдиханні. Перкуторно встановлено: над легенями коробковий звук, підчас аускультації на фоні подовженого видиху прослуховуються розсіяні сухі та дрібнопухірцеві вологі хрипи з обох боків. У крові виявлено: Hb -112 г/л, еритроцити - 3,2х10Л12/л, лейкоцити - 15,4х10Л9/л, лімфоцити - 72%. Який попередній діагноз? In a 3-month-old child against the background of subfebrile body temperature and rhinitis, there is pallor, cyanosis of the nasolabial triangle, pronounced shortness of breath of an expiratory nature, a swollen chest, a dry cough, the involvement of the auxiliary muscles of inhalation. Percussion established: a box-like sound over the lungs, during auscultation against the background of prolonged exhalation, scattered dry and small-vesicular wet rales are heard on both sides. Blood revealed: Hb -112 g/l, erythrocytes - 3.2x10L12/l, leukocytes - 15 ,4x10L9/l, lymphocytes - 72%. What is the previous diagnosis?

Гострий обструктивний бронхіт Acute obstructive bronchitis

Гострий (простий) бронхіт Acute (simple) bronchitis

Бронхіальна астма, приступний період Bronchial asthma, attack period

Двобічна пневмонія Bilateral pneumonia

Гострий бронхіоліт Acute bronchiolitis

38 / 150
Жінка 78 років, яка страждає на артеріальну гіпертензію, мала три епізоди раптової транзиторної втрати зору на ліве око. Під час аускультації сонних артерій уточках біфуркації з обох боків вислуховується шум. Яке додаткове дослідження найбільш доцільно призначити? A 78-year-old woman suffering from arterial hypertension had three episodes of sudden transient loss of vision in the left eye. During auscultation of the carotid arteries, there is a murmur on both sides of the bifurcation . What additional research is the most appropriate to prescribe?

МРТ головного мозку MRI brain

Люмбальна пункція Lumbar puncture

Нейросонографія Neurosonography

КТ головного мозку CT brain

Дуплексне сканування судин шиї Duplex scan of neck vessels

39 / 150
Чоловік 45 років скаржиться на втрату апетиту, запаморочення, виражену загальну слабкість, субфебрильну температуру. Шкіра бліда, синці на тулубі та кінцівках. ЗАК: Ер. - 2,9х1012/л, Нв-96 г/л, К.п. -0,98, ретикулоцити -0,02\%, Л. 2,9х109/л, Тр.-85х109/л, ШОЕ-45 мм/год. Яке додаткове дослідження є найбільш доцільним для верифікації діагнозу? A 45-year-old man complains of loss of appetite, dizziness, pronounced general weakness, subfebrile temperature. The skin is pale, bruises on the trunk and limbs. ZAK: Er. - 2, 9x1012/l, Hb-96 g/l, K.p.-0.98, reticulocytes -0.02\%, L. 2.9x109/l, Tr.-85x109/l, ESR-45 mm/h. What additional research is most appropriate for verifying the diagnosis?

Рівень сироваткового заліза Serum iron level

Вміст В12 в крові B12 content in blood

Коагулограма Coagulogram

Стернальна пункція Sternal puncture

Осмотична резистентність еритроцитів Osmotic resistance of erythrocytes

40 / 150
Жінка 28 років скаржиться на загальну слабкість, головний біль, підвищення температури тіла до 37-38 °С, незначний біль у горлі. Хворіє 3-й день. Об'єктивно встановлено: шкіра бліда, губи ціанотичні. Гіперемія ротоглотки з ціанотичним відтінком, набряклі язичок, піднебінні дужки, мигдалики. На поверхні мигдаликів - суцільні щільні білуваті з перламутровим відтінком нальоти, які знімаються шпателем з великим зусиллям, після їх видалення підлегла слизова оболонка кровоточить. Збільшені підщелепні лімфатичні вузли. Набряк шиї. Тахікардія. AT - 105/65 мм рт. ст. Який найбільш імовірний діагноз? A 28-year-old woman complains of general weakness, a headache, an increase in body temperature to 37-38 °C, a slight sore throat. She has been ill for the 3rd day. About objectively established: the skin is pale, the lips are cyanotic. Hyperemia of the oropharynx with a cyanotic shade, swollen tongue, palatal arches, tonsils. On the surface of the tonsils, there are continuous dense whitish plaques with a pearly tint, which are removed with a spatula with great effort, after their removal, the underlying mucous membrane bleeds . Enlarged submandibular lymph nodes. Swelling of the neck. Tachycardia. AT - 105/65 mm Hg. What is the most likely diagnosis?

Інфекційний мононуклеоз Infectious mononucleosis

Гострий лейкоз Acute leukemia

Дифтерія ротоглотки Oropharyngeal diphtheria

Ангіна Angina

Аденовірусна інфекція Adenovirus infection

41 / 150
Хлопчик 10 років, який спостерігається гематологом із приводу гемофілії, під час фізичних вправ пошкодив правий колінний суглоб. Під час огляду за годину після травми суглоб значно збільшений у розмірах, симптом балотування надколінка позитивний. Яку початкову тактику лікування слід обрати? A 10-year-old boy, who is being observed by a hematologist for hemophilia, injured his right knee joint during exercise. During the examination an hour after the injury, the joint is significantly enlarged, the patellar prolapse symptom is positive. What initial treatment tactics should be chosen?

Пункція суглоба Joint puncture

Артротомія Arthrotomy

Фізіотерапевтичне лікування Physiotherapy treatment

Спиртово-фураціліновий компрес Alcohol-furacilin compress

Гемостатична терапія, іммобілізація Hemostatic therapy, immobilization

42 / 150
Дівчинка 4-х років захворіла гостро після перенесеної 3 тижні тому стрептококової інфекції на шиї. З’явилися набряки на обличчя, нижніх кінцівок, головний біль, темний колір сечі (колір “кока-коли”). АТ -125/60 мм рт.ст. Аналіз сечі:білок – 3,3 %, питомна вага – 1012, лейкоцити 1-3 в п/з, еритроцити вкривають все п/з. Добова протеїнурія – 980 мг. Діурез – 550 мл. Загальний білок крові – 60 г/л. Альбуміни – 55%. Холестерин – 4,5 ммоль/л. Креатинін сироватки – 89 мкмоль/л. Який найбільший імовірний діагноз? A 4-year-old girl became acutely ill after suffering a streptococcal infection on her neck 3 weeks ago. Swelling appeared on the face, lower limbs, headache, dark urine ('Coca-Cola' color). Blood pressure -125/60 mm Hg. Urine analysis: protein - 3.3%, specific gravity - 1012, leukocytes 1-3 in p/z, erythrocytes cover all p/z. Daily proteinuria - 980 mg. Diuresis - 550 ml. Total blood protein - 60 g/l. Albumin - 55%. Cholesterol - 4.5 mmol/l. Serum creatinine - 89 μmol/l. What is the most likely diagnosis?

Пієлонефрит Pyelonephritis

Сечокам'яна хвороба Urolithiasis

Гостре ураження нирок Acute kidney injury

Гломерулонефрит, нефритичний синдром Glomerulonephritis, nephritic syndrome

Гломерулонефрит, нефротичний синдром Glomerulonephritis, nephrotic syndrome

43 / 150
Під час огляду пацієнтки 64-річного віку лікар діагностував ожиріння (індекс маси тіла -36 кг/м2, об'єм талії -118 см), артеріальну гіпертензію (170/105 мм рт. ст), порушення толерантності до вуглеводів. Під час обстеження виявлено підвищення рівню холестерину крові. Який діагноз хворої? During the examination of a 64-year-old female patient, the doctor diagnosed obesity (body mass index -36 kg/m2, waist circumference -118 cm), arterial hypertension ( 170/105 mm Hg), violation of tolerance to carbohydrates. During the examination, an increase in the level of blood cholesterol was detected. What is the patient's diagnosis?

Метаболічний синдром Metabolic syndrome

Артеріальна гіпертензія Hypertension

Гіпоталамічний синдром Hypothalamic syndrome

Цукровий діабет Diabetes

Аліментарне ожиріння Alimentary obesity

44 / 150
До опікового відділення доставлено потерпілого чоловіка 28 років зі циркулярними опіками тулуба та нижніх кінцівок. Визначте розмір опікової поверхні у потерпілого: A 28-year-old male victim was brought to the burn department with circular burns on the trunk and lower limbs. Determine the size of the victim's burn surface:

30% 30%

20% 20%

72%+ 72%+

36% 36%

45% 45%

45 / 150
Пацієнтка 40 років доставлена зі скаргами на переймоподібні болі внизу живота та надмірні кров'яні виділення зі статевих шляхів. Останні два роки менструації тривають до 16 днів, надмірні, із згустками, болісні. Підчас бімануального дослідження встановлена наявність фіброматозного вузла, що народжується. Оберіть правильну тактику: A 40-year-old female patient was brought in with complaints of spasm-like pains in the lower abdomen and excessive bleeding from the genital tract. For the last two years, menstruation has lasted up to 16 days, excessive, with with clots, painful. During the bimanual examination, the presence of a nascent fibromatous node was established. Choose the correct tactics:

Екстирпація матки без додатків Extirpation of the uterus without attachments

Вітамінотерапія Vitamin therapy

Гормональний гемостаз Hormonal hemostasis

Видалення фіброматозного вузла вагінально Removal of a fibromatous node vaginally

Надпіхвова ампутація матки без додатків Supravaginal amputation of the uterus without attachments

46 / 150
Жінка за 10 днів після травми ділянки стопи відчула утруднення під час відкриття рота. Наступного дня з важкістю могла приймати їжу, з'явилося напруження м'язів потилиці, спини, живота. Під час прийому в стаціонар на 3 день хвороби встановлено тонічне напруження всіх груп м'язів, генералізовані судоми кожні 10-15 хвилин. Який найбільш імовірний діагноз? 10 days after the injury to the foot, the woman felt difficulty opening her mouth. The next day, she could hardly take food, there was tension in the muscles of the back of the head and back , abdomen. During admission to the hospital on the 3rd day of the illness, tonic tension of all muscle groups was established, generalized convulsions every 10-15 minutes. What is the most likely diagnosis?

Геморагічний інсульт Hemorrhagic stroke

Правець Tetanus

Менінгітоенцефаліт Meningoencephalitis

Тетанія Tetany

Епілепсія Epilepsy

47 / 150
Улікаря-стоматолога 26 років під час обстеження виявили помірну гіперферментемію, позитивний тест на HBsAg. Його дружина 24 років не має щеплення від вірусного гепатиту В. Дитина 2 років належно вакцинована за віковим графіком. Яка профілактика вірусного гепатиту В повинна бути проведена у родині? A 26-year-old dentist was diagnosed with moderate hyperfermentemia during the examination, a positive HBsAg test. His 24-year-old wife is not vaccinated against viral hepatitis B. The 2-year-old child is properly vaccinated according to the age schedule. What prevention of viral hepatitis B should be carried out in the family?

Негайна вакцинація дружини при негативному тесті на HBsAg Immediate vaccination of wife with a negative HBsAg test

Призначення дружині та дитині специфічної хіміопрофілактики ненуклеозидними аналогами Prescribing specific chemoprophylaxis to the wife and child with non-nucleoside analogues

Призначення дружині специфічної хіміопрофілактики ненуклеозидними аналогами Prescribing specific chemoprophylaxis to the wife with non-nucleoside analogues

Призначення специфічного імуноглобуліну дружині та дитині Prescribing specific immunoglobulin to wife and child

Призначення специфічного імуноглобуліну дитині Prescribing specific immunoglobulin to a child

48 / 150
Проведено дослідження щодо встановлення зв'язку між ожирінням і інфарктом міокарду у медичних сестер. Протягом декількох років реєстрували і порівнювали частоту появи нових випадків інфаркту міокарда в групах та зміни ІМТ. Вкажіть тип епідеміологічного дослідження: A study was conducted to establish the relationship between obesity and myocardial infarction in nurses. For several years, the frequency of new cases of myocardial infarction in groups and changes in BMI were recorded and compared . Specify the type of epidemiological study:

Експериментальне дослідження Experimental study

Дослідження випадок-контроль Case-control study

Дослідження опис серії випадків Study description of case series

Когортне дослідження Cohort Study

Поперечне дослідження Cross-sectional study

49 / 150
На репрезентативній сукупності хворих на грип проведено вивчення зв'язку між температурою тіла і частотою пульсу. Розрахований коефіцієнт кореляції дорівнює +0,5. Яка характеристика сили і направленості зв'язку між ознаками, що вивчаються, є найбільш точною? The relationship between body temperature and pulse rate was studied on a representative population of flu patients. The calculated correlation coefficient is +0.5. What is the characteristic of the strength and directionality of 'is the connection between the signs being studied the most accurate?

Сильний прямий Strong straight

Слабкий прямий Weak direct

Середній зворотній Average inverse

Середній прямий Average straight

50 / 150
Хлопець 18 років перебуває на лікуванні в терапевтичному відділенні з приводу загострення хронічного бронхіту. З 6 років страждає епілепсією. На фоні підвищення температури до фебрильних цифр розвинувся епілептичний статус у вигляді тоніко-клонічних судом. Яка Ваша найперша тактика у цьому разі? An 18-year-old boy is being treated in the therapeutic department for an exacerbation of chronic bronchitis. He has been suffering from epilepsy since the age of 6. Against the background of an increase in temperature to febrile figures, status epilepticus has developed in the form of tonic-clonic seizures. What is your first tactic in this case?

Переведення до психіатричного стаціонару Transfer to a psychiatric hospital

Переведення до реанімаційного відділення Transfer to intensive care unit

Переведення до неврологічного відділення Transfer to the neurology department

Надання невідкладної допомоги на місці Providing emergency care on site

Організація консультації психіатра Organization of psychiatrist consultation

51 / 150
Чоловік ЗО років скаржиться на біль у гомілках обох ніг, який посилюється під час ходьби. Хворий змушений зупинятися через 150-200 м. Переміжна кульгавість з'явилась рік тому, відстань між зупинками скоротилася. Обидві ступні бліді, прохолодні. Пульс на тильних артеріях ступнів відсутній. Шкіра суха, на підошвах гіперкератоз, нігті потовщені, легко ламаються, матового кольору. На гомілках - ділянки облисіння. Яка стадія облітеруючого ендартеріїту у цього пацієнта? A 30-year-old man complains of pain in the lower legs of both legs, which worsens when walking. The patient is forced to stop after 150-200 m. Intermittent lameness appeared a year ago , the distance between stops has shortened. Both feet are pale, cool. There is no pulse on the back arteries of the feet. The skin is dry, hyperkeratosis on the soles, nails are thickened, break easily, matte color. There are areas of baldness on the lower legs. What is the stage of obliterating endarteritis in this patient?

IІІ стадія (декомпенсації) III stage (decompensation)

IV стадія (деструктивних змін) IV stage (destructive changes)

І стадія (функціональної компенсації) I stage (functional compensation)

II стадія (субкомпенсації) II stage (subcompensation)

52 / 150
Жінка 68 років була госпіталізована зі скаргами на головний біль у потиличній ділянці, виражену задишку в спокої, яка посилюється у положенні лежачи, та сухий кашель. Об'єктивно встановлено: положення ортопное, акроціаноз. Над легенями вислуховується ослаблене везикулярне дихання, середньо- і дрібно-пухірчасті вологі хрипи в нижніх відділах. Тони серця ослаблені, ритм галопу. Пульс - 102/хв., AT -210/110 мм рт. ст. Печінка виступає на 2 см з-під краю реберної дуги, чутлива під час пальпації. Набряки на гомілках. Які препарати потрібно призначити насамперед? A 68-year-old woman was hospitalized with complaints of a headache in the occipital region, pronounced shortness of breath at rest, which worsens when lying down, and a dry cough. Objectively established : orthopneic position, acrocyanosis. Weak vesicular breathing is heard above the lungs, medium and small-vesicular wet rales in the lower parts. Heart sounds are weakened, gallop rhythm. Pulse - 102/min., AT -210/110 mm Hg. Liver protrudes 2 cm from the edge of the costal arch, is sensitive during palpation. Swelling on the lower legs. What drugs should be prescribed first?

Цефтріаксон, лазолван в/в Ceftriaxone, Lazolvan IV

Левофлоксацин в/в, амброксол перорально IV levofloxacin, oral ambroxol

Нітрогліцерин, фуросемід в/в Nitroglycerin, IV furosemide

Сульфат магнію в/в, фуросемід в/м Magnesium sulfate IV, furosemide IV

Лабеталол в/в, фуросемід в/м Labetalol IV, Furosemide IV

53 / 150
Швидкою допомогою доставлена жінка з виниклим після затримки менструації переймоподібним болем в правій здухвинній ділянці, що іррадіює у пряму кишку, кров'янистими виділеннями зі статевих шляхів. Об'єктивно встановлено: частота серцевих скорочень - 100/хв., артеріальний тиск - 90/60 мм рт. ст. Шкіряні покриви бліді. Живіт болючий при пальпації, позитивний симптом Щоткіна-Блюмберга. Під час гінекологічного дослідження спостерігається: зсуви шийки болісні, праві придатки збільшені, болючі, заднє склепіння нависає, виділення кров'янисті. Поставте попередній діагноз: A woman was delivered by ambulance with cramp-like pain in the right pubic area, radiating into the rectum, blood discharge from the genital tract, which arose after the delay of menstruation. Objectively it was established: heart rate - 100/min, blood pressure - 90/60 mm Hg. The skin is pale. The abdomen is painful on palpation, a positive Stotkin-Blumberg symptom. During the gynecological examination, it is observed: cervical dislocations are painful, the right appendages are enlarged , painful, the posterior vault overhangs, the discharge is bloody. Make a preliminary diagnosis:

Гострий правобічний аднексит Acute right-sided adnexitis

Апоплексія правого яєчника Apoplexy of the right ovary

Апендицит Appendicitis

Позаматкова вагітність, що перервалася Interrupted ectopic pregnancy

Аборт в ходу Abortion in progress

54 / 150
У дівчинки 8 місяців під час огляду виявлений гучний інтенсивний систолічний шум над усією серцевою ділянкою з епіцентром у III-IV міжребер'ї зліва від груднини та систолічне тремтіння; II тон над легеневою артерією посиленої гучності. AT -110/70 мм рт. ст. Скарг немає. Під час рентгенологічного дослідження розміри і форма серцевої тіні не змінені. Який найімовірніший діагноз? In an 8-month-old girl, during the examination, a loud, intense systolic murmur was detected over the entire cardiac region with an epicenter in the III-IV intercostal space to the left of the sternum and systolic tremor; II tone over the pulmonary artery of increased volume. AT -110/70 mm Hg. There are no complaints. During the X-ray examination, the size and shape of the cardiac shadow have not changed. What is the most likely diagnosis?

Відкрита артеріальна протока Open ductus arteriosus

Стеноз гирла легеневої артерії Stenosis of the mouth of the pulmonary artery

Дефект міжпередсердної перетинки Atrial membrane defect

Дефект міжшлуночкової перетинки Defect of interventricular membrane

Коарктація аорти Coarctation of the aorta

55 / 150
Чоловіка 42 років, шахтаря, через 12 годин звільнено з-під завалу. Об'єктивно встановлено: обидві гомілки і ступні блідого кольору. Пульсації периферійних судин немає. Чутливість та пасивні рухи в суглобах відсутні. Доставлений із джгутами на обох кінцівках. Які невідкладні заходи першої лікарської допомоги необхідно провести для запобігання міоглобінурії та гострої ниркової недостатності? A 42-year-old man, a miner, was freed from the rubble after 12 hours. Objectively established: both lower legs and feet are pale in color. There is no pulsation of peripheral vessels. Sensitivity and passive joint movements are absent. Delivered with tourniquets on both limbs. What emergency first aid measures should be taken to prevent myoglobinuria and acute renal failure?

Зняти джгут, дезінтоксикаційна терапія Remove tourniquet, detoxification therapy

Джгут не знімати, гіпербарична оксигенація Do not remove tourniquet, hyperbaric oxygenation

Зняти джгут, гіпербарична оксигенація Remove tourniquet, hyperbaric oxygenation

Джгут не знімати, дезінтоксикаційна терапія Do not remove tourniquet, detoxification therapy

Джгут не знімати, ампутація кінцівок вище джгута Do not remove the tourniquet, amputation of limbs above the tourniquet

56 / 150
З дому надійшла дитина 3 років, у якої на тлі гіпертермічного синдрому, спричиненого грипом, протягом 50 хвилин тривають тоніко-клонічні генералізовані судоми. Терапія не проводилася. Для негайного лікування судомного синдрому треба: A 3-year-old child came from home with tonic-clonic generalized convulsions lasting 50 minutes against the background of hyperthermic syndrome caused by influenza. Therapy was not carried out. For immediate treatment of convulsive syndrome should:

Увести фенобарбітал внутрішньом'язово Inject phenobarbital intramuscularly

Увести лоразепам внутрішньовенно Inject lorazepam intravenously

Терміново накласти дитині холодні компреси Urgently apply cold compresses to the child

Призначити парацетамол у високих дозах Prescribe paracetamol in high doses

Увести кальцію глюконат внутрішньовенно Enter calcium gluconate intravenously

57 / 150
Чоловік 39 років скаржиться на задишку та стискальний біль за грудниною у стані спокою. 10 днів тому переніс грип. Об'єктивно встановлено: поза вимушена - сидить, нахиливши тулуб уперед, обличчя здутле, ціанотичне, здуті шийні вени. Межі серця значно розширені в обидві сторони, тони глухі, ритмічні, частота серцевих скорочень - 104/хв., частота дихальних рухів - 28/хв. На ЕКГ виявлено: зниження вольтажу зубців, конкордантні зміни сегменту ST. На рентгенограмі: кулеподібна тінь серця, ознаки застою в легенях. У крові виявлено: швидкість осідання еритроцитів - 38мм/год. Який імовірний діагноз? A 39-year-old man complains of shortness of breath and squeezing pain behind the sternum at rest. He had the flu 10 days ago. Objectively established: the posture is forced - he sits with his torso bent forward, the face is swollen, cyanotic, swollen neck veins. Heart borders are significantly expanded on both sides, sounds are dull, rhythmic, heart rate - 104/min., respiratory rate - 28/min. ECG revealed: decrease in voltage of the teeth, concordant changes in the ST segment. On the radiograph: a spherical shadow of the heart, signs of congestion in the lungs. In the blood, the sedimentation rate of erythrocytes is 38 mm/h. What is the probable diagnosis?

Ексудативний перикардит Exudative pericarditis

Вірусний міокардит Viral myocarditis

Нестабільна стенокардія Unstable angina

Ревматична гарячка Rheumatic fever

Дилатаційна кардіоміопатія Dilated cardiomyopathy

58 / 150
У чоловіка 35 років виникає задишка під час фізичного навантаження, він періодично втрачає свідомість. Аускультативно встановлено: систолічний шум у III міжребер'ї зліва від грудини. Під час УЗД серця встановлено: симетрична гіпертрофія ЛШ, його діастолічна дисфункція з деяким зменшенням порожнини. Спостерігається передньо-систолічний рух передньої стулки морального клапану. Яка патологія у хворого? A 35-year-old man develops shortness of breath during physical exertion, he periodically loses consciousness. Auscultation revealed: systolic murmur in the III intercostal space to the left of the sternum. During ultrasound of the heart was established: symmetric LV hypertrophy, its diastolic dysfunction with some reduction of the cavity. Ante-systolic movement of the front leaflet of the moral valve is observed. What is the patient's pathology?

Гіпертрофічна кардіоміопатія Hypertrophic cardiomyopathy

Дилятаційна кардіоміопатія Dilated cardiomyopathy

Рестриктивна кардіоміопатія Restrictive cardiomyopathy

Постінфарктний міокардіосклероз Postinfarction myocardial sclerosis

Стеноз вустя аорти Aortic stenosis

59 / 150
Жінка 68 років доставлена швидкою допомогою з носовою кровотечею. З анамнезу встановлено, що протягом 15 років вона хворіє на гіпертонічну хворобу, не лікувалась, AT -210/120 мм рт. ст. Періодичні носові кровотечі турбують декілька років. Об'єктивно встановлено: у носовій порожнині кров'янисті згустки, дихання вільнПід час видалення згустків з'явилась кровотеча з носової перегородки. Яку першу допомогу погрібно надати? A 68-year-old woman was delivered by ambulance with a nosebleed. From the anamnesis, it was established that she has been suffering from hypertension for 15 years, was not treated, AT -210/120 mm of mercury. Periodic nosebleeds have been bothering me for several years. It has been objectively established: there are bloody clots in the nasal cavity, breathing is free During the removal of the clots, bleeding appeared from the nasal septum. What first aid should be provided?

Задня тампонада Rear tamponade

Перев'язка сонної артерії на боці кровотечі Ligation of the carotid artery on the bleeding side

Тушування слизової носа розчином азотнокислого срібла або ваготілом Putting the nasal mucosa with a solution of silver nitrate or vagotyl

Відшарування слизової оболонки носової перегородки Defoliation of the mucous membrane of the nasal septum

Передня тампонада носа з судинозвужувальними препаратами, контроль за артеріальним тиском Anterior nasal tamponade with vasoconstrictor drugs, blood pressure control

60 / 150
У чоловіка 52 років, який страждає на постхолецистектомічний синдром, під час ретроградної холецистопанкреато-графії виявлено стеноз сфінктера Одді з помірною дилатацією загального жовчного протока і рефлюксом контраста у Вірсунгів проток. Конкрементів у протоках немає. Діагноз хронічного панкреатиту безсумнівний. Яка операція адекватна для корекції цієї патології? In a 52-year-old man suffering from post-cholecystectomy syndrome, stenosis of the sphincter of Oddi with moderate dilatation of the common bile duct and contrast reflux in the duct of Wirsung was revealed during retrograde cholecystopancreatography . There are no calculi in the ducts. The diagnosis of chronic pancreatitis is unquestionable. What operation is adequate to correct this pathology?

Продольна панкреатиконостомія Longitudinal PancreaticoStomy

Ендоскопічна папілосфінктеротомія Endoscopic papillosphincterotomy

Холсцисто-єюностомія Cholecystojejunostomy

Холедохо-дуоденостомія Choledocho-duodenostomy

Трансдуоденальна папілосфінктеропластика Transduodenal papillosphincteroplasty

61 / 150
Пацієнтка 37 років. Переведена до очного відділення із ЛОР відділення, у якому лікувалася з приводу гнійного пансинуситу. Об'єктивно встановлено: загальна слабість, температура тіла - 38,5 °С. Гострота зору правого ока - 0,6 (з анамнезу VIS OD=1,0). Повіки синюшні, набряклі, щільні. Хемоз кон'юнктиви. Екзофтальм. Нерухомість очного яблука. Репозиція ока затруднена. Поставте діагноз: The patient is 37 years old. She was transferred to the eye department from the ENT department, where she was treated for purulent pansinusitis. Objectively established: general weakness, body temperature - 38, 5 °С. The visual acuity of the right eye is 0.6 (from history VIS OD=1.0). The eyelids are bluish, swollen, dense. Chemosis of the conjunctiva. Exophthalmos. Immobility of the eyeball. Reposition of the eye is difficult. Make a diagnosis:

Ретробульбарна пухлина Retrobulbar tumor

Флегмона орбіти Phlegmon of the orbit

Ретробульбарна гематома Retrobulbar hematoma

Гострий блефарокон'юнктивіт Acute blepharoconjunctivitis

Псевдотумор орбіти Pseudotumor of the orbit

62 / 150
Дівчина 18 років скаржиться на тяжкість і розпирання в правому підребер'ї. Стан посилюється після їжі чи прийнятті но-шпи. Живіт мякий, безболісний, печінка не збільшена. АсТ - 35 ОД, АлТ - 40 ОД. Аналіз крові показав: лейкоцити - 5,2 Г/л, ШОЕ - 7 мм/год. УЗД встановлено: печінка не збільшена. Жовчний міхур натщесерце 68 мл, після жовчогінного сніданку на 45 хв. - 64 мл. Стінка не потовщена. Який найбільш імовірний діагноз? An 18-year-old girl complains of heaviness and distension in the right hypochondrium. The condition worsens after eating or taking no-shpa. The abdomen is soft, painless, the liver is not enlarged. AsT - 35 units, AlT - 40 units. Blood analysis showed: leukocytes - 5.2 g/l, ESR - 7 mm/h. Ultrasound revealed: the liver is not enlarged. Gallbladder on an empty stomach 68 ml, after a choleretic breakfast for 45 minutes. - 64 ml. The wall is not thickened. What is the most likely diagnosis?

Функціональна диспепсія Functional dyspepsia

Дискінезія жовчного міхура за гіпертонічним типом Gall bladder dyskinesia of the hypertensive type

Хронічний холецистит у стадії нестійкої ремісії Chronic cholecystitis in the stage of unstable remission

Дискінезія жовчного міхура за гіпотонічним типом Gall bladder dyskinesia of the hypotonic type

Синдром подразненого кишечника Teared Gut Syndrome

63 / 150
У новонародженого терміном гестації 31 тиждень наростає млявість, м'язова гіпотонія та пригнічення свідомості. Аналіз ліквору встановлено: підвищена кількість еритроцитів, білка та підвищений вміст глюкози. Який найбільш вірогідний діагноз? A newborn with a gestational age of 31 weeks develops lethargy, muscle hypotonia, and depression of consciousness. Analysis of the cerebrospinal fluid revealed: an increased number of erythrocytes, protein, and an increased glucose content. What is the most probable diagnosis?

Анемія Anemia

Внутрішньоутробна інфекція Intrauterine infection

Внутрішньочерепний крововилив Intracranial hemorrhage

Сепсис Sepsis

Менінгіт Meningitis

64 / 150
Чоловік 48 років звернувся до лікаря зі скаргами на біль у попереку. Хворіє 3 дні після переохолодження. Після огляду лікар поставив діагноз: гострий попереково-крижовий радикуліт. Які ліки слід призначити хворому? A 48-year-old man went to the doctor with complaints of lower back pain. He has been sick for 3 days after hypothermia. After the examination, the doctor diagnosed acute lumbosacral sciatica. What medicines should be prescribed to the patient?

Десенсибілізуючі Desensitizing

Вітаміни Vitamins

Нестероїдні протизапальні засоби Nonsteroidal anti-inflammatory drugs

Антибактеріальні Antibacterial

Кортикостероїди Corticosteroids

65 / 150
До відділення доставлено пацієнта у важкому стані: температура тіла - 36,5 °С, загальмований, риси обличчя загострені, язик сухий, шкірна складка не розправляється. Тахіпное до 28/хв., пульс - 126/хв., AT - 60/40 мм рт. ст., живіт безболісний. Під час госпіталізації спостерігалася: блювота фонтаном, водянисті випорожнення, що нагадували рисовий відвар. З епіданамнезу відомо: тиждень тому повернувся з Судану (Африка). Оберіть стартову терапію для невідкладної допомоги: A patient was brought to the department in a critical condition: body temperature - 36.5 °C, inhibited, facial features sharpened, tongue dry, skin fold does not straighten. Tachypnea to 28/min., pulse - 126/min., AT - 60/40 mm Hg, the abdomen is painless. During hospitalization, the following were observed: vomiting with a fountain, watery stools that resembled rice broth. It is known from the medical history: a week ago he returned from Sudan (Africa) Choose a starting therapy for emergency care:

Уведення свіжозамороженної плазми Introduction of fresh frozen plasma

Уведення антибіотиків Introduction of antibiotics

Уведення осмодіуретиків Introduction of osmodiuretics

Уведення колоїдних розчинів Introduction of colloidal solutions

Уведення кристалоїдних розчинів Introduction of crystalloid solutions

66 / 150
У сім'ї вживались в їжу овочеві (зокрема грибні) та фруктові консерви домашнього приготування. За 8 днів після чергового прийманні їжі удвох членів сім'ї з'явилися скарги на слабкість, послаблення зору, двоїння в очах та косоокість. Трохи згодом виникло утруднення мови, порушення ковтання. Після звернення до лікарні пацієнтів госпіталізували. Яке харчове отруєння виникло у цьому разі? The family used to eat home-made canned vegetables (especially mushrooms) and fruit. 8 days after the next meal, two members of the family appeared complaints of weakness, decreased vision, diplopia, and strabismus. A little later, difficulty in speaking, swallowing disorders appeared. After going to the hospital, the patients were hospitalized. What kind of food poisoning occurred in this case?

Ботулізм Botulism

Стафілококовий токсикоз Staphylococcal toxicosis

Афлотоксикоз Aflotoxicosis

Бактеріальна харчова токсикоінфекція Bacterial food poisoning

Сальмонельоз Salmonellosis

67 / 150
Пацієнтка 35 років скаржиться на сукровичні виділення з соска лівої молочної залози. Під час огляду, пальпації патології не виявлено. Для якого доброякісного захворювання є характерним цей симптом? A 35-year-old female patient complains of purulent discharge from the nipple of the left mammary gland. During the examination, palpation, no pathology was detected. For which benign disease is this symptom characteristic?

Фіброаденома молочної залози Breast fibroadenoma

Кіста молочної залози Breast gland cyst

Інфільтративний мастит Infiltrative mastitis

Масталгія Mastalgia

Внутрішньопротокова папілома Intraductal papilloma

68 / 150
Дівчинка народилася від І вагітності, І вагінальних пологів у терміні гестації 39 тижнів (маса - 3350 г, довжина тіла - 53 см). 17-річна мати дитини ВІЛ-позитивна. Стан дитини після народження задовільний. Коли можна провести вакцинацію БЦЖ цій дитині? A girl was born from I pregnancy and vaginal delivery at 39 weeks of gestation (weight - 3350 g, body length - 53 cm). 17-year-old mother of the child HIV -positive. The child's condition after birth is satisfactory. When can BCG vaccination be given to this child?

Після проби Манту After Mantoux test

На 3-тю добу On the 3rd day

Після уточнення ВІЛ-статусу дитини After clarifying the child's HIV status

На 1-шу добу For the 1st harvest

Не щеплювати в жодному разі Do not vaccinate in any case

69 / 150
У дитини 11 років спостерігаються періодичні закрепи від народження, здуття живота, зниження апетиту, інколи нудота, гіпотрофія м'язів. На іригограмі спостерігається звуження в ділянці прямої кишки із супрастенотичним розширенням ободової кишки вище цього місця. Під час аноманометрії аноректальний рефлекс негативний. Який метод діагностики буде найбільш інформативним для підтвердження діагнозу? An 11-year-old child has periodic constipation since birth, abdominal distension, decreased appetite, sometimes nausea, muscle atrophy. On the irrigogram, there is a narrowing in the area of the rectum with by suprastenotic extension of the colon above this place. During anomanometry, the anorectal reflex is negative. What diagnostic method will be the most informative to confirm the diagnosis?

Оглядова рентгенографія черевної порожнини Overview x-ray of the abdominal cavity

Повношарова біопсія стінки прямої та ободової кишки Full-layer biopsy of the wall of the rectum and colon

Термографія передньої черевної стінки Thermography of the anterior abdominal wall

Ультразвукове дослідження (УЗД) Ultrasound

Біохімічне дослідження крові Biochemical blood test

70 / 150
У дитини 3 років під час обстеження виявлено: висока температура, інтоксикація, блідість шкіри, геморагічні та некротичні елементи висипу на шкірі та слизових оболонках. У загальному аналізі крові виявлено: панцитопенія, агранулоцитоз, ретикулоцити відсутні, ШОЕ значно збільшена. Який імовірний діагноз? A 3-year-old child during the examination revealed: high temperature, intoxication, pale skin, hemorrhagic and necrotic elements of a rash on the skin and mucous membranes. A general blood test revealed : pancytopenia, agranulocytosis, reticulocytes are absent, ESR is significantly increased. What is the probable diagnosis?

Апластична анемія Aplastic anemia

Білково-дефіцитна анемія Protein deficiency anemia

В12-дефіцитна анемія B12 deficiency anemia

Залізодефіцитна анемія Iron deficiency anemia

Гемоглобінопатія Hemoglobinopathy

71 / 150
У дитини 13 років поєднана вада морального клапана з переважанням недостатності. На тлі гострої респіраторної вірусної інфекції (ГРВІ) підсилилися задишка та загальна слабкість, з'явилося відчуття стискання в грудях, сухий кашель. У легенях вислуховуються різнокаліберні вологі хрипи. Печінка біля краю реберної дуги. Який невідкладний стан розвинувся у дитини? A 13-year-old child has a combined defect of the moral valve with a predominance of insufficiency. Against the background of an acute respiratory viral infection (ARSI), shortness of breath and general weakness increased, a feeling of compression appeared in in the chest, dry cough. Wet rales of various caliber are heard in the lungs. Liver near the edge of the costal arch. What emergency condition has developed in the child?

Гостра лівошлуночкова серцева недостатність Acute left ventricular heart failure

Хронічна лівошлуночкова серцева недостатність Chronic left ventricular heart failure

Гостра тотальна серцева недостатність Acute total heart failure

Гостра правошлуночкова серцева недостатність Acute right ventricular heart failure

Гостра судинна недостатність Acute vascular insufficiency

72 / 150
Під час операції у жінки 67 років із приводу гострого деструктивного холециститу визначена наявність гнійного холангіта. Підчас ревізії холедоха не знайдені конкременти та ознаки стенозу дуоденального соска. Виконано холецистектомію. Як слід закінчити операцію? During the operation of a 67-year-old woman for acute destructive cholecystitis, the presence of purulent cholangitis was determined. No calculi or signs of duodenal nipple stenosis were found during choledochal revision. A cholecystectomy was performed. How should the operation be completed?

Виконати холедохоеюностомію Perform choledochojejunostomy

Накласти супрадуоденальний холедоходуоденоанастомоз Apply supraduodenal choledochoduodenal anastomosis

Накласти гепатікоєюностомію Apply hepaticojejunostomy

Виконати трансдуоденальну холедоходуоденостомію Perform transduodenal choledochoduodenostomy

Виконати зовнішнє дренування холедоха Perform external choledochal drainage

73 / 150
Під час проведення епідеміологічного дослідження на території міста згідно висунутою гіпотезою вивчалися причино-наслідкові зв'язки виникнення нових випадків захворювань залежно від визначених факторів ризику. Який вид епідеміологічного дослідження був використаний? During an epidemiological study on the territory of the city, according to the proposed hypothesis, the cause-and-effect relationships of the occurrence of new cases of diseases depending on the identified risk factors were studied. What type of epidemiological study was used?

Клінічний Clinical

Описовий Descriptive

Експериментальний Experimental

Квазі-експериментальний Quasi-experimental

Аналітичний Analytic

74 / 150
Чоловік 52 років страждає на виражену задишку під час фізичного навантаження, непродуктивний кашель. Хворіє 8 місяців. Палить ЗО років. Під час аускультації в легенях з обох боків вислуховуються крепітуючі хрипи - «тріск целофану». ЧДД - 26/хв., SpO2 - 92%. Під час спірометрії виявили: помірні порушення функції зовнішнього дихання за рестриктивним типом. Запідозрили інтерстиційний процес. Який метод дослідження найбільш ефективно допоможе верифікувати діагноз та прогноз? A 52-year-old man suffers from pronounced shortness of breath during physical exertion, a non-productive cough. He has been ill for 8 months. He has been a smoker for 30 years. During auscultation in the lungs, crackles are heard on both sides wheezing - 'crack of cellophane'. HRD - 26/min., SpO2 - 92%. During spirometry, moderate violations of the function of external breathing according to the restrictive type were found. An interstitial process was suspected. What research method will most effectively help verify the diagnosis and prognosis?

Торакоскопія з біопсією Thoracoscopy with biopsy

Фібробронхоскопія Fibrobronchoscopy

Рентгенографія органів грудної клітки X-ray of chest organs

Комп'ютерна томографія Computed tomography

Бактеріологічне дослідження харкотиння Bacteriological examination of sputum

75 / 150
Чоловік 62 років. Протягом останніх 3 років без видимих причин стала зростати безпорадність, зниженя пам'яті. У психічному стані встановлено: правильно називає своє прізвище, ім'я, по батькові, дату народження; вік сказати не можДезорієнтований у місцевості, у часі. Пам'ять знижена на поточні та віддалені події. Інтелект знижений. Порушені вищі коркові функції з явищами афазії, апраксії, агнозії. Критика відсутня. Визначіть провідний патогенетичний механізм формування описаного стану: The man is 62 years old. Over the past 3 years, for no apparent reason, helplessness and memory loss began to increase. In his mental state, it was established: correctly pronounces his last name, first name , patronymic, date of birth; age cannot be said. Disoriented in place, in time. Memory is reduced for current and distant events. Intelligence is reduced. Disrupted higher cortical functions with phenomena of aphasia, apraxia, agnosia. No criticism. Identify the leading pathogenetic mechanism of formation of the described state:

Нейродегенеративний Neurodegenerative

Надлишок серотоніну в синаптичній щілині Excess serotonin in the synaptic cleft

Надлишок ацетилхолінтрансферази Acetylcholine transferase excess

Дефіцит норадреналіну Noradrenaline deficiency

Гіперліпідемія Hyperlipidemia

76 / 150
Чоловік 45 років скаржиться на біль у внутрішній поверхні гомілки і стегна справа. Хворіє протягом 5 діб, коли після фізичного навантаження з'явився біль у гомілці, який поступово поширився на стегно. Об'єктивно встановлено: у проекції великої підшкірної вени на гомілці та стегні спостерігається гіперемія шкіри і припухлість. При пальпації виявлено: ущільнення та болючість. Набряки правої нижньої кінцівки відсутні, пульсація артерій на стопах визначається. Яка найбільш імовірна патологія, що зумовлює таку картину? A 45-year-old man complains of pain in the inner surface of the lower leg and thigh on the right. He has been sick for 5 days, when after physical exertion pain appeared in the lower leg, which gradually spread on the thigh. Objectively established: skin hyperemia and swelling are observed in the projection of the large saphenous vein on the lower leg and thigh. Palpation revealed: compaction and tenderness. There is no swelling of the right lower limb, pulsation of the arteries on the feet is determined. What is the most likely pathology that causes such a picture?

Висхідний тромбофлебіт Ascending thrombophlebitis

Гострий лімфангіт Acute lymphangitis

Гострий тромбоз підколінної та стегнових вен Acute thrombosis of popliteal and femoral veins

Спонтанний розрив великої підшкірної вени Spontaneous rupture of the great saphenous vein

Гострий тромбоз підколінної та стегнової артерій Acute thrombosis of popliteal and femoral arteries

77 / 150
Жінка 65 років госпіталізована до реанімаційного відділення в коматозному стані. Хворіє на цукровий діабет II типу 10 років. Останні 2 тижні спостерігалися виражена поліурія, полідипсія. Глюкоза сироватки крові - ЗО ммоль/л, pH артеріальної крові - 7,3. Осмолярність плазми крові - 350 мОсм/л. Хворій встановлено діагноз: діабетична гіперосмолярна кома. Який основний патогенетичний механізм коми? A 65-year-old woman was hospitalized in the intensive care unit in a comatose state. She has been suffering from type II diabetes for 10 years. In the last 2 weeks, severe polyuria and polydipsia were observed. Blood serum glucose - 3 mmol/l, arterial blood pH - 7.3. Blood plasma osmolarity - 350 mOsm/l. The patient was diagnosed with diabetic hyperosmolar coma. What is the main pathogenetic mechanism of coma?

Дегідратація Dehydration

Гіпонатріємія Hyponatremia

Підвищення клубочкової фільтрації Increase glomerular filtration

Підвищення канальцевої реабсорбції Increasing tubular reabsorption

Гіпокоагуляція Hypocoagulation

78 / 150
До гінекологічного відділення звернулася жінка 25 років із скаргами на підвищення температури тіла до 38,6 °С, біль внизу живота, дизуричні розлади. Захворіла 3 доби тому, коли після штучного аборту з'явилися вказані симптоми. Під час гінекологічного дослідження виявлено: шийка матки циліндрична, вічко закритТіло матки дещо збільшене, болюче, м'якДодатки матки не пальпуються. Виділення гнійно-кров'яні. За результатами аналізу крові встановлено: лейкоцитоз зі зміщенням формули крові вліво, прискорена ШОЕ. Який діагноз найбільш імовірний? A 25-year-old woman came to the gynecology department with complaints of an increase in body temperature to 38.6 °C, pain in the lower abdomen, dysuric disorders. She became ill 3 days ago, when after an artificial abortion, the indicated symptoms appeared. During a gynecological examination, it was found: the cervix is cylindrical, the eye is closed. The body of the uterus is slightly enlarged, painful, soft. blood formulas to the left, accelerated ESR. What is the most likely diagnosis?

Гострий сальпінгоофорит Acute salpingo-oophoritis

Гострий цистит Hostry cystitis

Піосальпінкс Piosalpinx

Гострий ендометрит Acute endometritis

Гострий ендоцервіцит Acute endocervicitis

79 / 150
Чоловік 54 років звернувся до сімейного лікаря зі скаргами на утруднення дихання, слабкість. Останні два тижні турбував біль та набряк правої нижньої кінцівки. Ці симптоми відчуває вперше в житті, раніше на обліку у лікаря не перебував. AT -110/70 мм рт. ст, ЧСС -96/хв. Який діагностичний метод має вирішальне значення? A 54-year-old man consulted a family doctor with complaints of difficulty breathing, weakness. For the past two weeks, he has been troubled by pain and swelling of the right lower limb. He is experiencing these symptoms for the first time in his life, I have not been registered with a doctor before. AT -110/70 mm Hg, heart rate -96/min. What diagnostic method is crucial?

Мультиспіральна КТ ангіографія Multispiral CT angiography

Функція зовнішнього дихання External breathing function

Електрокардіографія Electrocardiography

Ехокардіографія Echocardiography

Рентгенографія органів грудної клітки X-ray of chest organs

80 / 150
У породіллі на 12-ту добу післяпологового періоду раптово підвищилася температура тіла до 38,2 °С. Спостерігається загальна слабкість, біль в ділянці правої молочної залози протягом 1 доби. Під час огляду встановлено: молочна залоза напружена, гаряча, у правому верхньому квадранті пальпується інфільтрат, болючий, щільної консистенції. Який діагноз найбільш імовірний? On the 12th day of the postpartum period, the mother's body temperature suddenly rose to 38.2 °C. There was general weakness, pain in the area of the right mammary gland for 1 day . During the examination, it was found: the mammary gland is tense, hot, in the right upper quadrant a painful, dense infiltrate is palpable. What is the most likely diagnosis?

Лактостаз Lactic Stasis

Лактаційний мастит Lactation mastitis

Мастопатія Mastopathy

Аномалії розвитку молочних залоз Mammary gland development anomalies

Пухлина молочної залози Breast tumor

81 / 150
Чоловікові 61 року зроблено холецистектомію 7 діб тому. Супутнє захворювання -цукровий діабет. Яку дієту слід призначити хворому? A 61-year-old man underwent a cholecystectomy 7 days ago. The concomitant disease is diabetes mellitus. What diet should the patient be prescribed?

Дієта № 15 Diet #15

Дієта № 5а Diet No. 5a

Дієта № 7 Diet #7

Дієта № 9 Diet #9

Дієта № 10 Diet #10

82 / 150
Дитина народилася на 8 місяці вагітності. У неї встановлено: мікроцефалію, катаракту, ваду серця. Мати дитини на 2-му місяці вагітності хворіла; було нетривале підвищення температури тіла до 37,5 °С, збільшення лімфатичних вузлів та дрібноплямистий висип на обличчі, тулубі й кінцівках, який пройшов без залишкових явищ. Який найбільш імовірний попередній діагноз у дитини і матері? The child was born at the 8th month of pregnancy. She was diagnosed with: microcephaly, cataracts, heart defect. The child's mother was ill in the 2nd month of pregnancy; there was a short-term increase in body temperature up to 37.5 °C, an increase in lymph nodes and a small papular rash on the face, trunk and limbs, which passed without residual effects. What is the most likely preliminary diagnosis for the child and the mother?

Герпетична інфекція Herpes infection

Токсоплазмоз Toxoplasmosis

Цитомегаловірусна інфекція Cytomegalovirus infection

Хламідійна інфекція Chlamydial infection

Краснуха Krasnukha

83 / 150
Вагітна в терміні 35 тижнів, стан тяжкий. Скаржиться на головний біль, порушення зору, миготіння «мушок» перед очима. Під час огляду виявлено: загальний набряк, AT -180/120 мм рт. ст. Раптово з'явилися фібрилярні посмикування м'язів обличчя, тонічні судоми. Дихання припинилося. За 1 хвилину дихання відновилося. З рота виділилася значна кількість піни. Амнезія. Протеінурія - 7 г/л. Який імовірний діагноз? Pregnant at 35 weeks, the condition is serious. She complains of a headache, impaired vision, flickering of 'flies' in front of the eyes. During the examination, it was found: general edema, AT -180/120 mm Hg. Fibrillary twitching of the facial muscles, tonic convulsions suddenly appeared. Breathing stopped. Breathing resumed within 1 minute. A significant amount of foam was released from the mouth. Amnesia. Proteinuria - 7 g/l. What is the probable diagnosis?

Еклампсія Eclampsia

Передеклампсія тяжкого ступеню Severe preeclampsia

Гіпертонічний криз Hypertensive crisis

Епілепсія Epilepsy

Черепно-мозкова травма Tranio-cerebral injury

84 / 150
Чоловікові 69 років проведена правостороня нефректомія. Інтраопераційна одномоментна крововтрата - 800 мл, після якої протягом 40 хвилин спостерігалась гіпотонія. Надалі розвинулась анурія. Укажіть вірогідну причину анурії: A 69-year-old man underwent a right-sided nephrectomy. Intraoperative one-time blood loss was 800 ml, after which hypotension was observed for 40 minutes. Later, anuria developed. Specify the probable cause of anuria:

Обтурація сечового катетера Urinary catheter obstruction

Синдром дисемінованого внутрішньосудинного згортання Disseminated intravascular coagulation syndrome

Тампонада сечового міхура Bladder tamponade

Гостра ниркова недостатність Acute renal failure

Хронічна ниркова недостатність Chronic renal failure

85 / 150
Чоловік 58 років скаржиться на болі в правій здухвинній ділянці, нечасті до 4-6 разів на добу, випорожнення, підвищення температури тіла до 37,5-38 °С. Об'єктивно встановлено: стан середньої важкості зниженого харчування, шкіра та слизові оболонки бліді. AT - 100/60 мм рт. ст., PS - 98/хв. Пальпується болючий інфільтрат у правій здухвинній ділянці. Дослідження крові показало: еритроцити - 2,5х10д12/л, НЬ — 80 г/л, лейкоцити -11,2 10х9/л, еозинофіли - 3%, паличкоядерні - 6%, сегментоядерні - 68%, моноцити - 7%, лімфоцити -14%, загальний білок - 61 г/л, альбуміни - 47%, глобуліни - 53%, С-реактивний білок - 24. Під час рентгенографічного дослідження з пасажем барію по тонкій кишці виявлений запальний стеноз термінального відділу тонкої кишки з депонуванням барію у дилятованій петлі тонкої кишки. Поставлено діагноз: хвороба Крона з ураженням термінального відділу тонкої кишки, стенозуюча форма середньої важкості. Яку групу препаратів слід призначити насамперед? A diagnosis was made: Crohn's disease with damage to the terminal part of the small intestine, stenotic form of medium severity. Which group of drugs should be prescribed first?

Ферменти Enzymes

Антибіотики Antibiotics

Сульфаніламіди Sulfonamides

Пробіотики Probiotics

Спазмолітики (як запасний варіант - сульфаніламіди) Spasmolytics (as a spare option - sulfonamides)

86 / 150
У жінки 65 років, яка вживала воду з криниці, поступово підвищилась температура тіла до 39,9 °С. Об'єктивно встановлено: загальмована, язик вкритий сіро-бурим нальотом із відбитками зубів по краях, на шкірі живота поодинокі розеоли, живіт здутий. Реакція Відаля - 1:400. Який із препаратів етіотропної хіміотерапії необхідно призначити? A 65-year-old woman, who drank water from a well, gradually increased her body temperature to 39.9 °C. It was objectively established: she was inhibited, her tongue was covered with gray brown plaque with teeth marks on the edges, isolated roseolae on the skin of the abdomen, the abdomen is swollen. Vidal's reaction - 1:400. Which of the etiotropic chemotherapy drugs should be prescribed?

Цефазолін Cefazolin

Ніфуроксазид Nifuroxazide

Ципрофлоксацин Ciprofloxacin

Пеніцилін Penicillin

Еритроміцин Erythromycin

87 / 150
Пацієнтка 20 років проходить лікування з приводу анемії (гемоглобін - 72 г/л). Півтора роки тому після мимовільного викидня у терміні 16 тижнів та крововтрати, спостерігається зниження пам'яті, утомлюваність, втрату апетиту, сухість шкіри, ламкість нігтів, набряклість, порушення менструальної функції. Об'єктивно встановлено: артеріальний тиск - 80/55 мм рт. ст., пульс - 54/хв., зріст -168 см, вага - 48 кг, гіпоплазія статевих органів. Призначення якого з наведених препаратів буде найбільш доречним? A 20-year-old patient is being treated for anemia (hemoglobin - 72 g/l). One and a half years ago, after an involuntary miscarriage at 16 weeks and blood loss, a decrease in fatigue, loss of appetite, dry skin, brittle nails, swelling, violation of menstrual function Objectively established: blood pressure - 80/55 mm Hg, pulse - 54/min, height -168 cm, weight - 48 kg, genital hypoplasia. Which of the following drugs would be the most appropriate?

Інфліксимаб Infliximab

Гідрокортизон Hydrocortisone

Гідроксихлорохін Hydroxychloroquine

Метотрексат Methotrexate

Імуноглобулін лОД. ини нормальний Immunoglobulin is normal

88 / 150
У водія під час клінічного обстеження у сечі виявлено підвищений вміст кальцієвих солей фосфорної кислоти. Його раціон харчування містить житній та пшеничний хліб, макаронні вироби, вершкове масло, олія, картопляне пюре, молоко, сир, кава, чай, відвар шипшини, кисіль зі смородини. Енергоцінність раціону відповідає енерговитратам. Що потрібно обмежити у раціоні? During the clinical examination, the driver was found to have an elevated content of calcium salts of phosphoric acid in his urine. His diet includes rye and wheat bread, pasta, butter, oil, mashed potatoes, milk, cheese, coffee, tea, rosehip decoction, currant jelly. The energy value of the diet corresponds to energy expenditure. What should be limited in the diet?

Молоко і сир Milk and cheese

Каву та чай Coffee and tea

Кисіль зі смородини Currant jelly

Макаронні вироби і хліб Pasta and bread

Відвар шипшини і кисіль Decoction of rose hips and jelly

89 / 150
До лікаря в ОКЛ звернулась жінка 35 років зі скаргами на біль за грудиною, утруднення проходження твердої та рідкої їжі стравоходом, підвищену слюнотечу, зригування їжі, похудання. Такі симптоми турбують близько року після перенесеного психоемоційного стресу. Який попередній діагноз? A 35-year-old woman came to the doctor in OKL with complaints of pain behind the sternum, difficulty passing solid and liquid food through the esophagus, increased salivation, regurgitation of food, weight loss. Such symptoms bother about a year after the psycho-emotional stress. What is the previous diagnosis?

Атрезія стравоходу Esophageal atresia

Песляопіковий рубцевий стеноз Post-acute cicatricial stenosis

Рак стравоходу Cancer to the Strawwalker

Дивертикул стравоходу Esophageal diverticulum

Ахалазія стравоходу Esophageal achalasia

90 / 150
Під час проведення медичного огляду учнів середнього та старшого шкільного віку лікарі визначали відповідність біологічного розвитку та календарного віку за такими критеріями: щорічне збільшення довжини тіла, осифікація кісток кисті, кількість постійних зубів. Який додатковий показник розвитку у цьому віковому періоді найбільш імовірно мають включити лікарі? During the medical examination of middle and high school students, doctors determined the correspondence of biological development and calendar age according to the following criteria: annual increase in body length, ossification of hand bones, number permanent teeth. What additional developmental indicator should doctors most likely include in this age period?

Розвиток вторинних статевих ознак Development of secondary sexual characteristics

Обвід грудної клітки Chest Circumference

Маса тіла Body weight

М'язова сила кисті Hand muscle strength

Життєва ємність легень Vital lung capacity

91 / 150
У жінки 32 років за 6 годин після фіброфагогастроскопії підвищилася температура тіла до 39 °С, болі за грудиною, сухість у роті, напростає задишка. Під час обстеження виявлено позитивний симптом Романова-Гірке та підшкірну емфізему в надключичному просторі зліва. Який попередній діагноз? 6 hours after fibrophagogastroscopy, a 32-year-old woman's body temperature rose to 39 °C, pains behind the sternum, dry mouth, shortness of breath. During the examination, a positive Romanov-Ghirke symptom and subcutaneous emphysema in the supraclavicular space on the left. What is the previous diagnosis?

Емпієма плеври Empyema of the pleura

Гострий медіастиніт Acute mediastinitis

Пневмонія Pneumonia

Піопневмоторакс Pyopneumothorax

Гострий абсцесе легені Acute lung abscess

92 / 150
У дитини 11 років з цукровим діабетом 1-го типу на фоні гострої респіраторної вірусної інфекції посилилася спрага, з'явилася нудота та блювота. У видихуваному повітрі різкий запах ацетону. Очі запалі. Дихання шумне, аритмічне, типу Кусмауля. ЧСС - 128/хв., глухість серцевих тонів. Діагностована кстоацидотична кома. Укажіть провідний патогенетичний механізм розвитку невідкладного стану у дитини: In an 11-year-old child with type 1 diabetes, against the background of an acute respiratory viral infection, thirst increased, nausea and vomiting appeared. The exhaled air has a sharp smell of acetone . The eyes are inflamed. Breathing is noisy, arrhythmic, Kussmaul type. Heart rate - 128/min., deafness of heart sounds. Diagnosed xtoacidotic coma. Specify the leading pathogenetic mechanism of the development of an emergency condition in a child:

Надлишкова продукція інсуліну Excess insulin production

Гіперактивність центральної нервової системи Hyperactivity of the central nervous system

Активація симпато-адреналової системи Activation of sympatho-adrenal system

Недостатність функції кори наднирників Adrenal cortex insufficiency

Абсолютна інсулінова недостатність Absolute insulin deficiency

93 / 150
У хворого 10 років діагностовано геморагічний васкуліт, шкіряна форма. Одним з основних лікувальних заходів є тривала преднізолонотерапія. З якою метою застосовується ця терапія? A 10-year-old patient was diagnosed with hemorrhagic vasculitis, cutaneous form. One of the main treatment measures is long-term prednisolone therapy. What is the purpose of this therapy?

Підвищення синтезу простагландинів Increased synthesis of prostaglandins

Підвищення синтезу антитіл Increasing synthesis of antibodies

Зменшення синтезу простагландинів Decreased synthesis of prostaglandins

Зменшення синтезу патологічних імунних комплексів Decreased synthesis of pathological immune complexes

Як замістна терапія As replacement therapy

94 / 150
Хлопчик 6 років поступив із скаргами на набряки обличчя і нижніх кінцівок. Об'єктивно встановлено: у нижніх відділах легень притуплення, ослаблене дихання. Набряклість передньої черевної стінки. Асцит. AT - 90/50 мм рт. ст. Загальний аналіз сечі виявив: білок - 4,2 г/л, лейкоцити - 5-6 у п/з, еритроцити - 2-3 у п/з. Загальний білок крові - 48,6 г/л, холестерин - 8,2ммоль/л. Попередній діагноз: гострий гломерулонефрит. Укажіть клінічний варіант захворювання: A 6-year-old boy was admitted with complaints of swelling of the face and lower limbs. Objectively established: dullness in the lower parts of the lungs, weakened breathing. Swelling of the anterior abdominal wall. Ascites AT - 90/50 mm Hg. General analysis of urine revealed: protein - 4.2 g/l, leukocytes - 5-6 in p/z, erythrocytes - 2-3 in p/z. Total blood protein - 48 .6 g/l, cholesterol - 8.2 mmol/l. Preliminary diagnosis: acute glomerulonephritis. Specify the clinical variant of the disease:

Нефротичний синдром Nephrotic Syndrome

Нефротичний синдром з гематурією і гіпертензією Nephrotic syndrome with hematuria and hypertension

Нефритичний синдром Jade Syndrome

Сечовий синдром Urinary syndrome

Нефротичний синдром з гематурією Nephrotic syndrome with hematuria

95 / 150
У сухому піщаному ґрунті був знайдений труп чоловіка 45-50 років, шкіра якого буро-брунатного кольору, пергаментної щільності. Під час обстеження трупа встановлено: травматичні ушкодження відсутні, довжина тіла 172 см, внутрішні органи зменшені, сухі, безформні, окремі органи плівчасті. В якому стані трупних змін перебуває труп? The corpse of a 45-50-year-old man was found in the dry sandy soil, the skin of which is brownish-brown in color, with parchment density. During the examination of the corpse, it was established: there are no traumatic injuries, body length 172 cm, internal organs are reduced, dry, shapeless, some organs are membranous. What state of cadaveric changes is the corpse in?

Жировоску Fat Wax

Гниття Rot

Штучного консервування Artificial preservation

Торф'яного дублення Peat tanning

Муміфікації Mummifications

96 / 150
Лікар оглядає недоношену новонароджену дитину. Об'єктивно спостерігалося одиничне апное тривалістю до 5 секунд. Якою основною функціональною особливістю дихальної системи можна пояснити приступи апное у недоношеної дитини? The doctor examines a premature newborn baby. A single apnea lasting up to 5 seconds was objectively observed. What main functional feature of the respiratory system can explain apnea attacks in a premature baby?

Недостатня дренажна та захисна функція бронхів Insufficient drainage and protective function of the bronchi

Схильність слизових оболонок до набряку та гіперсекреції Propensity of mucous membranes to edema and hypersecretion

Вузкість просвіту дихальних шляхів Narrowness of airway lumen

М'якість хрящів трахеї та бронхів Softness of tracheal and bronchial cartilages

Функціональна незрілість дихального центру Functional immaturity of the respiratory center

97 / 150
На прохідників під час виконання робіт у гірничих виробітках діє мінеральний пил у вигляді аерозолів дезінтеграції. Для визначення рівня небезпеки виникнення професійних пилових захворювань вивчали хімічні та фізичні властивості пилу. Яка із властивостей пилу визначає загалом глибину його проникнення в дихальні шляхи? Pedestrians are exposed to mineral dust in the form of disintegration aerosols during mining operations. To determine the level of danger of occupational dust diseases, the chemical and physical properties of dust were studied. What from the properties of dust determines in general the depth of its penetration into the respiratory tract?

Електрозарядженість Electrical charge

Вміст діоксиду кремнію Content of silicon dioxide

Дисперсність Variance

Розчинність Solubility

Форма пилових часток Shape of dust particles

98 / 150
Дівчинка вагою 3100 г народилася від третьої вагітності в домашніх умовах. На 3-й день мати помітила дрібні крововиливи у пупковій ранці, а також «червоні цятки» на склерах очей. Мати на обліку по вагітності у допологовому періоді не перебувала. Вигодовує дитину грудним молоком за потребою. Під час фізикального обстеження спостерігається субкон'юнктивальні геморагії. Під час лабораторного дослідження встановлено концентрацію гемоглобіну -155 г/л. Група крові у матері В (III) Rh-позитивна, у дитини - А (II) Rh-негативна. Яка тактика з метою попередження прогресування цього стану у дитини буде найбільш доречною? A girl weighing 3100 g was born from the third pregnancy at home. On the 3rd day, the mother noticed small hemorrhages in the umbilical wound, as well as 'red spots' on the sclera eyes. The mother was not registered for pregnancy in the prenatal period. She feeds the child with breast milk as needed. During the physical examination, subconjunctival hemorrhages are observed. During the laboratory examination, the hemoglobin concentration was found to be -155 g/l. The mother's blood group is B (III ) is Rh-positive, the child has A (II) Rh-negative. Which tactic to prevent the progression of this condition in the child would be the most appropriate?

Уведення вітаміну К Vitamin K administration

Перехід на штучне вигодовування залізовмісними сумішами Transition to artificial feeding with iron-containing mixtures

Призначити пероральний прийом препаратів заліза Prescribe oral iron preparations

Уведення імуноглобуліну антирезус RhO(D) лОД. ини Introduction of anti-rhesus RhO(D) immunoglobulin

Переливання еритроцитарної маси Red blood cell transfusion

99 / 150
Чоловік 38 років захворів 2 тижні тому, з'явився кашель, слабкість, підвищилася температура до 38,0 °С. Стан різко погіршився до кінця 1-го тижня, коли з'явилися остуда, проливний піт, надвечір температура підвищилася до 39,0 °С. За 2 дні до госпіталізації у хворого під час кашлю виділилася велика кількість смердючого харкотиння з кров'ю, після чого стан пацієнта поліпшився. Пульс - 80/хв., частота дихання - 20/хв., температура тіла -37,6 °С. Які зміни можливі на рентгенограмі грудної порожнини? A 38-year-old man fell ill 2 weeks ago, cough, weakness appeared, the temperature rose to 38.0 °C. The condition worsened sharply by the end of the 1st week , when chills, profuse sweat appeared, in the evening the temperature rose to 39.0 ° C. 2 days before hospitalization, the patient coughed up a large amount of foul-smelling sputum with blood, after which the patient's condition improved. Pulse - 80/ min., respiratory rate - 20/min., body temperature -37.6 °C. What changes are possible on the X-ray of the chest cavity?

Однорідна округла тінь у легеневому полі Uniform round shadow in lung field

Наявність порожнини з горизонтальнім рівнем рідини Presence of a cavity with a horizontal liquid level

Тінь у нижньому відділі з косою верхньою межею Shadow in lower section with oblique upper border

Зміщення середостіння в бік гомогенної тіні Displacement of mediastinum towards homogeneous shadow

Затемнення частки легені Obscuration of a lobe of the lung

100 / 150
Мама скаржиться на висипи та інтенсивний свербіж шкіри у дитини 11 років, яка захворіла на 1-му році життя. Шкірні покриви сухі. Переважно на згинах колінних і локтьових суглобів, передньої поверхні шиї, обличчі видні вогнища ліхенізації з великою кількістю ексоріацій, плоскі вузлові висипання, кольору нормальної шкіри, лущення, гіперпігментація. Поставте імовірний діагноз: The mother complains of rashes and intense itching of the skin in an 11-year-old child who got sick in the 1st year of life. The skin is dry. Mainly on the bends of the knee and elbow joints , on the front surface of the neck, on the face, foci of lichenization with a large number of excoriations, flat nodular rashes, normal skin color, peeling, hyperpigmentation are visible. Make a probable diagnosis:

Хронічна істинна екзема Chronic true eczema

Кропив'янка Hives

Червоний плоский лишай Red lichen planus

Атопічний дерматит Atopic dermatitis

Псоріаз Psoriasis

101 / 150
У чоловіка 22 років з'явились сильні болі в нижній частині спини, які іррадіюють у п'яткові кістки, внутрішню поверхню стегон, спостерігається підвищення температури до 38,2 °С. Затри тижні відчув важкорухомість у хребті, болі в ілеосакральних суглобах. Рентгенологічно виявлено: розширення щілини ілеосакрального з'єднання, склерозування спінальних зв'язків. Про яке захворювання слід думати? A 22-year-old man developed severe pain in the lower back, radiating to the heel bones, the inner surface of the thighs, an increase in temperature up to 38.2 °С. Three weeks ago, I felt difficulty moving in the spine, pain in the ileosacral joints. X-ray revealed: widening of the ileosacral joint gap, sclerosing of the spinal ligaments. What disease should we think about?

Оперізуючий лишай Zingles

Киста сакрального відділу хребта Cyst of the sacral spine

Анкілозуючий спондилоартрит Ankylosing spondylitis

Грижа спинального каналу Spinal canal herniation

Міжреберна невралгія Intercostal neuralgia

102 / 150
Дівчина 19 років скаржиться на виражений біль у поперековій ділянці, більше справа, підвищення температури тіла до 38,5 °С, часте сечовиділення, симптоми з'явились 5 днів тому після переохолодження (похід у гори). Об'єктивно встановлено: шкіра бліда, волога. Пульс - 100/хв., AT - 120/65 мм рт. ст. Живіт м'який, симптом Пастернацького позитивний більше справа. ЗАК виявив: еритроцити - 3,5х10Л12/л, НЬ -115 г/л, лейкоцити - 10х10д9/л, ШОЕ - 22 мм/год. ЗАС виявив: щільність -1018 г/л, білок - 0,099 г/л, лейкоцити - 35-45 в п/з, переважають сегментоядерні, еритроцити -3-4 в п/з. велика кількість слизу, бактерій. Який попередній діагноз? A 19-year-old girl complains of severe pain in the lumbar region, more on the right, an increase in body temperature to 38.5 °C, frequent urination, symptoms appeared 5 days ago therefore, after hypothermia (hiking in the mountains). Objectively established: the skin is pale, moist. Pulse - 100/min., AT - 120/65 mm Hg. Abdomen is soft, Pasternacki's symptom is positive more on the right. ZAK found: erythrocytes - 3.5x10L12/l, Hb -115 g/l, leukocytes - 10x10d9/l, ESR - 22 mm/h. ZAS revealed: density -1018 g/l, protein - 0.099 g/l, leukocytes - 35-45 in p/z, segmentonuclear cells predominate, erythrocytes -3-4 in p/z. a large amount of mucus, bacteria. What is the preliminary diagnosis?

Хронічний пієлонефрит Chronic pyelonephritis

Туберкульоз нирок Kidney tuberculosis

Гострий пієлонефрит Acute pyelonephritis

Гострий гломерулонефрит Acute glomerulonephritis

Хронічний гломерулонефрит Chronic glomerulonephritis

103 / 150
На зупинці автобуса чоловік похилого віку схопився за серце, а потім втратив свідомість. Декілька разів глибоко вдихнув з інтервалом у 8-10 секунд, а потім перестав дихати. Шкіра бліда, холодна. Пульс на сонних артеріях не визначається. Зіниці розширені, реагують на світло. Діагностована раптова коронарна смерть. З чого варто почати серцево-легеневу реанімацію? At the bus stop, an elderly man clutched his heart and then lost consciousness. He took several deep breaths at intervals of 8-10 seconds and then stopped breathing. Skin pale, cold. The pulse on the carotid arteries is not determined. Pupils are dilated, react to light. Sudden coronary death is diagnosed. Where should cardiopulmonary resuscitation be started?

Непрямий масаж серця Indirect heart massage

Прийом Вальсальви Valsalva Reception

Массаж каротидного синуса Carotid sinus massage

Трахеостомія Tracheostomy

Штучна вентиляція легень Artificial lung ventilation

104 / 150
Для клінічно здорової дитини 6 місяців, яка перебуває на природному вигодовуванні потрібно призначити перший прикорм. Яку страву найбільш доцільно використати у цьому разі? For a clinically healthy child of 6 months who is on natural feeding, it is necessary to prescribe the first complementary food. Which dish is the most appropriate to use in this case?

Гречана каша Buckwheat porridge

Овочеве пюре Vegetable puree

Ячнева каша Barley porridge

Печиво Cookie

Терте яблуко Grated apple

105 / 150
У чоловіка хворого на гіпертонію поступово розвинулися ознаки серцевої недостатності: задишка, вологі хрипи у легенях, збільшення печінки. Яке інструментальне дослідження має більшу діагностичну цінність для встановлення варіанту дисфункції міокарда? A man with hypertension gradually developed signs of heart failure: shortness of breath, wet rales in the lungs, enlarged liver. Which instrumental examination has a greater diagnostic value for establishing the variant of myocardial dysfunction ?

Фонокардіографія Phonocardiography

Ехокардіографія Echocardiography

Електрокардіографія Electrocardiography

Тетраполярна реографія Tetrapolar rheography

Коронарографія Coronary Angiography

106 / 150
У дівчинки 11 років (в анамнезі часті ангіни) захворювання мало поступовий розвиток. З'явились дратівливість, неуважність, гіперкінези. Підчас обстеження виявлено порушення координації рухів, гіпотонію м'язів, гіпсррефлексію. Для якого захворювання характерні такі зміни з боку центральної нервової системи? In an 11-year-old girl (with a history of frequent angina), the disease had a gradual development. Irritability, inattention, hyperkinesis appeared. During the examination, movement coordination disorders, hypotonia 'muscles, hyporeflexia. What disease is characterized by such changes on the part of the central nervous system?

Вузликовий поліартеріїт Polyarteritis nodosa

Вегето-судинна дисфункція Vegeto-vascular dysfunction

Гостра ревматична лихоманка Acute rheumatic fever

Системна склеродермія Systemic Scleroderma

Системний червоний вовчак Systemic lupus erythematosus

107 / 150
Чоловік 40 років протягом 10 років страждає на хронічний гломерулонефрит. Скаржиться на блювоту, судоми м'язів нижніх кінцівок. AT -180/120 мм рт. ст., креатинін у сироватці крові - 770 мкмоль/л, швидкість клубочкової фільтрації - 5 мл/хв. Протягом останніх двох днів діурез знизився до 400 мл на добу. Яка лікувальна тактика показана для цього хворого? A 40-year-old man has been suffering from chronic glomerulonephritis for 10 years. He complains of vomiting, muscle spasms of the lower limbs. AT -180/120 mm Hg, creatinine in blood serum - 770 μmol/l, glomerular filtration rate - 5 ml/min. Over the past two days, diuresis has decreased to 400 ml per day. What therapeutic tactics are indicated for this patient?

Плазмаферез Plasmapheresis

Гемосорбція Hemosorption

Гемодіаліз Hemodialysis

Гемофільтрація Hemofiltration

Сорбенти Sorbents

108 / 150
Пацієнт 18 років хворіє з раннього дитинства. Під час обстеження встановлено: НЬ -110 г/л, еритроцити - 3,9х10д12/л, КП - 0,8, лейкоцити - б,0х10д9/л; ШОЕ - ЗО мм/год. Коагулограма показала: протромбіновий індекс - 95%, ретракція кров'яного згустка - 50%, час згортання крові - за 40 хвилин не відбулось, тривалість кровотечі - 3 хв. Які механізми складають основу патогенезу цього захворювання? An 18-year-old patient has been sick since early childhood. During the examination, it was found: Hb -110 g/l, erythrocytes - 3.9x10d12/l, CP - 0.8 , leukocytes - b,0x10d9/l; ESR - 30 mm/h. The coagulogram showed: prothrombin index - 95%, retraction of the blood clot - 50%, blood clotting time - did not occur for 40 minutes, duration of bleeding - 3 minutes. What mechanisms form the basis of the pathogenesis of this disease?

Наявність специфічних антитіл до ендотеліальних стінок судин Presence of specific antibodies to the endothelial walls of vessels

Дефіцит вітаміну С Vitamin C deficiency

Екзогенний дефіцит заліза Exogenous iron deficiency

Недостатність у крові антигемофільного глобуліну А Antihemophilic globulin A deficiency in the blood

Імунне пригнічення кісткового мозку Bone marrow immune suppression

109 / 150
Чоловік 25 років до психіатричної лікарні надходить впершЗбуджений, говорить швидко й голосно,активно жестикулює. Без упину жартує, сміється, з обличчя не сходить посмішка. З лікарем розмовляє на «ти», пропонує йому зіграти в карти, залицяється до медсестер, запрошує їх на побачення. Розповідає, що останні 2 доби не спав, проте зовсім не відчуває втоми. Який препарат доцільно використати? A 25-year-old man comes to a psychiatric hospital for the first time. Excited, speaks quickly and loudly, gestures actively. Jokes nonstop, laughs, a smile never leaves his face. He talks to the doctor in 'you' offers him to play cards, flirts with the nurses, invites them on a date. He says that he hasn't slept in the last 2 days, but he doesn't feel tired at all. What drug should be used?

Снодійний Sleepy

Антипсихотичний Antipsychotic

Антидепресант Antidepressant

Седативний Sedative

Ноотропний Nootropics

110 / 150
У хлопця 18 років за 2 тижня після ангіни з'явилися набряки під очима, на гомілках, підвищився артеріальний тиск, зменшилась кількість сечі і змінився її колір. Загальний аналіз сечі виявив: білок -1,9 г/л, еритроцити - покривають усе поле зору, лейкоцити -10-12 у полі зору. Для якого захворювання характерні такі симптоми? 2 weeks after angina, an 18-year-old boy developed swelling under his eyes, on his lower legs, increased blood pressure, decreased the amount of urine and changed its color. General analysis urine revealed: protein -1.9 g/l, erythrocytes - covering the entire field of vision, leukocytes -10-12 in the field of vision. What disease is characterized by such symptoms?

Гострий гломерулонефрит Acute glomerulonephritis

Токсичне ураження нирок Toxic kidney damage

Тубуло-інтерстиційний нефрит Tubulo-interstitial nephritis

Амілоїдоз нирок Kidney amyloidosis

Гострий пієлонефрит Acute pyelonephritis

111 / 150
Жінка 55 років скаржиться на пронос, лущення і пігментацію відкритих ділянок тіла (шиї, рук та стоп), дратівливість і занепокоєння. З анамнезу відомо, що основним продуктом харчування для неї є кукурудза. Салатні овочі і бобові жінка вживає рідко, м'ясо і рибу в їжу не вживає зовсім. Про яке захворювання йдеться? A 55-year-old woman complains of diarrhea, peeling and pigmentation of open areas of the body (neck, hands and feet), irritability and anxiety. From the anamnesis it is known that the main food for her there is corn. The woman rarely eats salad vegetables and legumes, she does not eat meat and fish at all. What disease is it about?

Цинга Цинга

Пелагра Пелагра

Псоріаз Psoriasis

Дерматит Свіфта Swift's dermatitis

Бері-бері Ever since

112 / 150
Чоловік 69 років протягом декількох останніх років спостерігав прогресуюче порушення сечовипускання. Протягом останньої доби зазначив відсутність сечовипускання, постійний розпираючий біль над лоном. Які діагностичні заходи слід запропонувати у цьому разі? A 69-year-old man has had a progressive urinary incontinence for the past several years. During the last day, he noted the absence of urination, constant throbbing pain over the pubic area. What diagnostic measures should be offered in this case ?

Внутрішньовенна урографія Intravenous urography

Катетеризація сечового міхура Bladder catheterization

Уретротомія Urethrotomy

Радіоізотопна ренографія Radioisotope renography

Оглядова урографія Review urography

113 / 150
Чоловік 35 років збитий автомобілем, ударився головою об асфальт. Об'єктивно встановлено: стан важкий, без свідомості, у комі, дихання уривчасте, шумне, на больові подразники не реагує. У ділянці лоба велика рана м'яких тканин голови, видно фрагменти кістки, витікає мозковий детрит, є кровотеча. Визначте лікувальну тактику на догоспітальному етапі: A 35-year-old man was hit by a car, hit his head on the asphalt. Objectively established: the condition is serious, unconscious, in a coma, breathing is intermittent, noisy, on painful stimuli does not respond. In the forehead area, there is a large wound of the soft tissues of the head, bone fragments are visible, brain detritus flows out, there is bleeding. Determine the treatment tactics at the pre-hospital stage:

Зупинка кровотечі, асептична пов'язка Stop bleeding, aseptic dressing

Зашивання рани Stitched Wounds

Протинабрякова терапія Anti-edema therapy

Видалення мозкового детриту Removing brain debris

Видалення фрагментів кісток Removing bone fragments

114 / 150
Хлопець 20 років пройшов курс поліхіміотерапії за схемою ВАМП з приводу гострого лімфобластного лейкозу. Яка морфологічна картина кісткового мозку може свідчити про настання ремісії? A 20-year-old boy underwent a course of polychemotherapy according to the VAMP scheme for acute lymphoblastic leukemia. What morphological picture of the bone marrow can indicate the onset of remission?

Вміст бластних клітин до 1% Content of blast cells up to 1%

Вміст бластних клітин до 10% Content of blast cells up to 10%

Відсутність бластних клітин No blast cells

Вміст бластних клітин до 15% Content of blast cells up to 15%

Вміст бластних клітин до 5% Content of blast cells up to 5%

115 / 150
Чоловік 64 років поступив до інфекційного відділення. Під час обстеження діагностовано вірусний гепатит А. З анамнезу відомо, що він страждає на цукровий діабет 2-го типу, з приводу чого він протягом останніх 2 років отримував глібенкламід 20 мг на добу. Глікемія натще -15,6 ммоль/л. Яка повинна бути подальша тактика лікування? A 64-year-old man was admitted to the infectious disease department. During the examination, viral hepatitis A was diagnosed. From the anamnesis, it is known that he suffers from type 2 diabetes, due to why he has been receiving glibenclamide 20 mg per day for the past 2 years. Fasting blood glucose - 15.6 mmol/l. What should be the further tactics of treatment?

Перевести хворого на інсулінотерапію Transfer the patient to insulin therapy

Додатково призначити бігуаніди Additionally assign biguanides

Перевести хворого на похідні сульфонілсечовини III генерації Transfer the patient to sulfonylureas of the III generation

Додатково призначити акарбозу Additionally prescribe acarbose

Збільшити дозу глібенкламіду до 20 мг на добу Increase the dose of glibenclamide to 20 mg per day

116 / 150
Жінка 22 років із підозрою на системний червоний вовчак скаржиться на «летючий» біль у суглобах рук і ніг, підвищення температури до 38,5-39 °С упродовж 3 тижнів, задишку, серцебиття, слабкість. Об'єктивно встановлено: еритема на щоках та носі. У крові виявлено: НЬ - 90 г/л, тромбоцити - 135х10д9/л, ШОЕ - 43 мм/год. У сечі виявлено: білок - 2,66 г/л, еритроцитів - 8-10 в п/з. Виявлення яких антитіл у пацієнтки найбільш значиме для постановки діагнозу? A 22-year-old woman with suspicion of systemic lupus erythematosus complains of 'flying' pain in the joints of the hands and feet, an increase in temperature to 38.5-39 °C for 3 weeks, shortness of breath, palpitations, weakness. Objectively established: erythema on the cheeks and nose. In the blood: Hb - 90 g/l, platelets - 135x10d9/l, ESR - 43 mm/h. In the urine: protein - 2 .66 g/l, erythrocytes - 8-10 in p/z. The detection of which antibodies in the patient is most significant for making a diagnosis?

До 2-х спіральної нативної ДНК Up to 2-helix native DNA

До тромбоцитів To platelets

До фосфоліпідів To phospholipids

Кріоглобуліни Cryoglobulins

Ревматоїдний фактор Rheumatoid factor

117 / 150
Жінка 29 років звернулась до центру планування сім'ї та репродуктивного здоров'я зі скаргами на безпліддя протягом 6 років подружнього життя. Місячні з 14 років по 3-4 дні, кількість виділень незначна, нерегулярні. Об'єктивно встановлено: гірсутизм, галакторея. Базальна температура -монофазна. Яка найбільш імовірна причина жіночого безпліддя? A 29-year-old woman applied to the family planning and reproductive health center with complaints of infertility during 6 years of married life. Monthly from 14 years to 3-4 days, the number of secretions is insignificant, irregular. Objectively established: hirsutism, galactorrhea. Basal temperature is monophasic. What is the most likely cause of female infertility?

Матковий фактор Maternal Factor

Нез'ясованого генезу Of unknown origin

Імунологічні фактори Immunological factors

Трубні, перітонеальні фактори Tubular, peritoneal factors

Ендокринні фактори Endocrine factors

118 / 150
Вагітна в терміні 34 тижні доставлена до пологового будинку через кровотечу зі статевих шляхів, що з'явилася під час сну. Біль не турбує. Кровотеча близько 300 мл. Який імовірний діагноз? A 34-week pregnant woman was taken to the maternity ward because of bleeding from the genital tract that appeared during sleep. The pain does not bother her. Bleeding is about 300 ml. What probable diagnosis?

Розрив судин пуповини Rupture of umbilical cord vessels

Передчасне відшарування нормально розташованої плаценти Premature detachment of a normally located placenta

Кровоточива ерозія Bleeding erosion

Передлежання плаценти Placenta previa

Рак шийки матки Cervical cancer

119 / 150
У хворого 2 років із рецидивуючим бронхо-легеневим захворюванням, гіпотрофією, установлений діагноз: муковісцидоз. Яке дослідження повинно підтвердити клінічний діагноз? A 2-year-old patient with recurrent broncho-pulmonary disease, hypotrophy, diagnosed with cystic fibrosis. What research should confirm the clinical diagnosis?

Кількість хлоридів у поті Number of chlorides in sweat

Наявність білку в сечі Presence of protein in urine

Вміст 2-фракції гамма-глобулінів Content of 2-fraction of gamma-globulins

Рівень кальцію в крові Calcium level in blood

Рівень лужної фосфатази в сироватці Serum alkaline phosphatase level

120 / 150
Під час огляду новонародженого, який народився в сідничному передлежанні, педіатр установив обмеження пасивного відведення правого стегна і клацання у разі його відведення, асиметрію пахових і стегнових сідничних складок. Яка ознака достовірно свідчить про наявність вродженого вивиху стегна у дитини? During the examination of a newborn who was born in the breech presentation, the pediatrician established a limitation of passive abduction of the right hip and clicking in the event of its abduction, asymmetry of the inguinal and femoral buttock folds. Which does the sign reliably indicate the presence of congenital dislocation of the hip in the child?

Достовірної ознаки немає There is no valid sign

Клацання у разі відведення стегна Click when abducting hip

Асиметрія пахових і стегнових сідничних складок Asymmetry of inguinal and femoral buttock folds

Обмеження пасивного відведення стегна Limit passive hip abduction

Народження в сідничному передлежанні Birth in breech

121 / 150
Чоловік 22 років лежить у ліжку, високо підвівши голову, не відчуває незручності в такій позі. У бесіду вступає неохоче, реагує на шепітну мову, відповіді однослівні. Обличчя байдуже, гіпомімічне, чоло наморщене, губи витягнуті трубочкою. Рухається дуже мало, часто надовго застигає в малозручній позі. Такий стан розвинувся поступово, протягом тижня без видимих причин. Діагностуйте психічний стан хворого: A 22-year-old man lies in bed with his head raised high, he does not feel discomfort in this position. He enters the conversation reluctantly, responds to whispered speech, answers are monotonous. The face is indifferent , hypomimic, forehead wrinkled, lips stretched out with a tube. Moves very little, often freezes for a long time in an uncomfortable position. This condition developed gradually, over a week without apparent reasons. Diagnose the mental state of the patient:

Екзогенний ступор Exogenous Stupor

Депресивний субступор Depressive substupor

Апатичний субступор Apathetic substupor

Психогенний ступор Psychic Stupor

Кататонічний субступор Catatonic substupor

122 / 150
Дитина 1,5 років у розпал гострої респіраторної вірусної інфекції (ГРВІ) їла волоський горіх. Раптово закашлялася. Дитина ціанотична. Періодичний сухий кашель зі свистом на видиху продовжується. Стан дитини легкого ступеню важкості, дихальна недостатність. Вкорочення перкуторного звуку, жорстке дихання, сухі і вологі хрипи, бронхоспазм на видиху над правою легенею. Якою буде подальша тактика? A 1.5-year-old child ate a walnut in the midst of an acute respiratory viral infection (ARSI). He suddenly coughed. The child is cyanotic. Periodic dry cough with wheezing continues. The child's condition is of a mild degree of severity, respiratory insufficiency. Shortening of the percussion sound, hard breathing, dry and moist wheezing, bronchospasm on exhalation over the right lung. What will be the further tactics?

Бронхоскопія Bronchoscopy

Комп'ютерна томограма грудної клітки Computer tomography of the chest

Оглядова рентгенографія грудної клітки Comprehensive chest X-ray

Ультразвукове дослідження (УЗД) живота та грудної клітки Ultrasound examination (ultrasound) of abdomen and chest

Оглядова рентгенографія живота Abdominal overview radiography

123 / 150
Після багаторазової обробки операційного інструменту слабоконцентрованим розчином дезинфікуючої рідини у медичної сестри поступово з'явилось запалення шкіри долоневої поверхні кистей рук. Об'єктивно встановлено: долонні поверхні набряклі, на них визначається дифузна еритема без чітких меж, на фоні якої є поодинокі папульозні висипання. Шкірні алергічні проби негативні. Який попередній діагноз? After repeated processing of the surgical instrument with a weakly concentrated solution of disinfectant liquid, the nurse gradually developed inflammation of the skin of the palmar surface of the hands. Objectively established: the palmar surfaces are swollen, on they are defined as diffuse erythema without clear borders, against the background of which there are isolated papular rashes. Skin allergy tests are negative. What is the previous diagnosis?

Токсидермія Toxidermy

Екзема Eczema

Хімічний опік Chemical burn

Контактний дерматит Contact dermatitis

Сенсибілізаційний дерматит Sensitizing dermatitis

124 / 150
Пацієнтці 37 років, після збору анамнезу та огляду виставлено діагноз: дифузна форма мастопатії обох молочних залоз. Який інструментальний метод дослідження найбільш інформативний у цьому разі? The patient is 37 years old, after taking an anamnesis and examination, the diagnosis was made: diffuse form of mastopathy of both mammary glands. Which instrumental research method is the most informative in this case?

Біопсія молочної залози Breast biopsy

УЗД молочних залоз Ultrasound of mammary glands

Дуктографія Ductography

Мамографія Mammography

Пневмокістографія Pneumocystography

125 / 150
У чоловіка з хронічним гнійним отитом виник сильний головний біль, блювання, підвищилася температура тіла. Наявні менінгеальні симптоми. Вогнищева неврологічна симптоматика відсутня. Які першечергові заходи для ведення хворого? A man with chronic purulent otitis has a severe headache, vomiting, and increased body temperature. Meningeal symptoms are present. There are no focal neurological symptoms. What are the first-line measures for managing the patient?

Призначення рентгенографії черепа Cranial x-ray appointment

Госпіталізація та діагностична люмбальна пункція Hospitalization and diagnostic lumbar puncture

Амбулаторне спостереження за хворим Ambulatory monitoring of the patient

Призначення протизапальних ліків Prescription of anti-inflammatory drugs

Консультація отоларинголога Otolaryngologist consultation

126 / 150
Жінка 56 років із надмірною масою тіла скаржиться на біль у колінних суглобах, який посилюється під час ходіння, особливо по східцях, під час довгого стояння на ногах. Хворіє 5 років. Об'єктивно встановлено: колінні суглоби деформовані, набряклі, підчас руху -болючі. Рентгенограма суглобів показала: суглобова щілина звужена, субхондральний склероз, краєві остеофіти. Який діагноз найбільш імовірний? A 56-year-old overweight woman complains of pain in the knee joints, which worsens when walking, especially on stairs, when standing on her feet for a long time. Patient 5 years old. It has been objectively established: the knee joints are deformed, swollen, painful during movement. X-ray of the joints showed: narrowed joint space, subchondral sclerosis, marginal osteophytes. What is the most likely diagnosis?

Ревматичний артрит Rheumatoid arthritis

Остеоартроз Osteoarthrosis

Реактивний артрит Reactive Arthritis

Ревматоїдний артрит Rheumatoid arthritis

Подагра Gout

127 / 150
Дочка викликала сімейного лікаря до своєї матері 77 років. У хворої онкологічне захворювання в термінальній стадії. Вона скаржиться на постійні болі та інші виснажливі симптоми. До якого закладу охорони здоров'я найдоцільніше госпіталізувати цю жінку? The daughter called the family doctor to see her 77-year-old mother. The patient has an oncological disease in the terminal stage. She complains of constant pain and other debilitating symptoms. To which health care institution 'Is it most expedient to hospitalize this woman?

Онкологічний диспансер Oncological dispensary

Територіальний центр соціального обслуговування пенсіонерів Territorial center of social service for pensioners

Соціальний пансіонат Social boarding house

Хоспіс Хоспіс

Багатопрофільна територіальна лікарня Multidisciplinary territorial hospital

128 / 150
Під час проведення медичного огляду хворого чоловіка лікар звернув увагу на перекорнеальну ін'єкцію судин склери очей («віночок фіалково-блакитного кольору»), кон'юктивіт, хейлоз, ангулярний стоматит, гіпертрофічний глосит. Яке із наведених захворювань можна припустити? During the medical examination of a sick man, the doctor drew attention to the percorneal injection of the vessels of the sclera of the eyes ('violet-blue corolla'), conjunctivitis, cheilosis , angular stomatitis, hypertrophic glossitis. Which of the following diseases can be assumed?

Гіповітаміноз рибофлавіну Riboflavin hypovitaminosis

Гіповітаміноз ціанокобаламіну Cyanocobalamin hypovitaminosis

Гіповітаміноз тіаміну Thiamin hypovitaminosis

Гіповітаміноз піридоксину Pyridoxine hypovitaminosis

Гіповітаміноз ніацину Niacin hypovitaminosis

129 / 150
Громадянин заключив декларацію з сімейним лікарем приватного закладу. Для проходження чергового медичного огляду, лікар дав направлення хворому на біохімічний аналіз крові. Хто є платником медичної послуги у цьому разі? The citizen made a declaration with the family doctor of a private institution. To undergo another medical examination, the doctor referred the patient for a biochemical blood test. Who is the payer of the medical service in this case?'

Пацієнт Patient

За рахунок місцевого бОД. жету At the expense of the local database

Спеціальний фонд лікарні Special hospital fund

Благодійні внески Charity contributions

НСЗУ (Пацієнт або НСЗУ) NSZU (Patient or NSZU)

130 / 150
Чоловік 35 років надійшов до клініки з явищами хірургічного сепсису, джерелом якого, ймовірно, був великий карбункул лопаткової області. Під час обстеження виявлені вторинні гнійні вогнища в печінці та правій легені. Назвіть фазу хірургічного сепсису: A 35-year-old man came to the clinic with symptoms of surgical sepsis, the source of which was probably a large carbuncle in the scapular area. During the examination, secondary purulent foci were found in the liver and right lungs. Name the phase of surgical sepsis:

Септикопіемія Septicopemia

Термінальна Terminal

Септицемія Septicemia

Гнійно-резорбтивна лихоманка Suppurative resorptive fever

Токсемія Toxemia

131 / 150
До лікаря звернувся чоловік 36 років зі скаргами на пекучий загрудинний біль, що зазвичай виникає за 1-1,5 години після вживання їжі. Зазначає, що біль підсилюється в горизонтальному положенні. Підчас ЕГДС виявлено 2 вогнищевих ураження нижньої третини слизової оболонки стравоходу до 5 мм у межах однієї складки. Яка тактика лікаря буде найбільш доречною? A 36-year-old man consulted a doctor with complaints of burning chest pain, which usually occurs 1-1.5 hours after eating. He notes that the pain intensifies in in a horizontal position. During EGD, 2 focal lesions of the lower third of the esophageal mucosa up to 5 mm within one fold were detected. What tactics of the doctor would be the most appropriate?

Призначення кларитроміцину Clarithromycin Prescription

Хірургічне лікування Surgical treatment

Призначення нснаркотичних анальгетиків Prescription of non-narcotic analgesics

Призначення інгібіторів протонної помпи Prescription of proton pump inhibitors

132 / 150
Чоловік 38 років скаржиться на періодичну висипку в ділянці бороди й вусів, що супроводжується помірною болючістю. Хворіє впродовж 2 років. Об'єктивно встановлено: у ділянці бороди та вусів на фоні гіперемії та помірної інфільтрації спостерігається множинні згруповані пустульозні елементи, дрібні ерозії та гнійні кірки. На яке супутнє захворювання слід обстежити пацієнта насамперед? A 38-year-old man complains of a periodic rash in the area of the beard and mustache, which is accompanied by moderate pain. He has been ill for 2 years. Objectively established: in the area of the beard and mustache against the background of hyperemia and moderate infiltration, multiple grouped pustular elements, small erosions and purulent crusts are observed. For which concomitant disease should the patient be examined first?

Дисбіоз кишечника Intestinal dysbiosis

Гепатит С Hepatitis C

Цукровий діабет Diabetes

Захворювання щитоподібної залози Thyroid disease

Туберкульозна інфекція Tuberculosis infection

133 / 150
Хвора 20 років скаржиться на затримку менструації на десять днів, порушення менструальної функції відмічає впершСтатеве життя регулярнОб'єктивно встановлено: стан задовільний, живіт м'який, безболісний. AT - 120/80 мм рт. ст., пульс - 72/хв. Під час УЗД виникла підозра на наявність прогресуючої трубної вагітності. Яка тактика лікаря жіночої консультації? A 20-year-old patient complains of a ten-day delay in menstruation, the first thing she notices is a violation of menstrual function. Sex life is regular. Objectively established: the condition is satisfactory, the abdomen is soft, painless. AT - 120/80 mm Hg, pulse - 72/min. During the ultrasound, there was a suspicion of the presence of a progressive tubal pregnancy. What are the tactics of the doctor in the women's consultation?

Терміново госпіталізувати хвору для уточнення діагнозу Urgently hospitalize the patient to clarify the diagnosis

Провести пункцію черевної порожнини через заднє склепіння піхви Perform a puncture of the abdominal cavity through the posterior vault of the vagina

Видати лікарняний лист та направити хвору на визначення титру ХГ сечі Issue a sick leave and send the patient to determine the titer of hCG urine

Провести обстеження за тестами функціональної діагностики Conduct an examination using functional diagnostic tests

Рекомендувати повторну явку за 1 тиждень для контрольного УЗД Recommend repeat appearance in 1 week for control ultrasound

134 / 150
Чоловік 25 років надійшов через 2 години після дорожньо-транспортної аварії зі скаргами на постійний інтенсивний біль у правій паховій ділянці та у симфізі тазу, неможливість самостійно пересуватися. Під час огляду встановлено: положення хворого вимушене - поза «жаби» (позитивний симптом Волковича). Садна й синці на шкірі в проекції симфізу та горизонтальної гілки лобкової кістки праворуч, виражений набряк м'яких тканин. Позитивний симптом «прилиплої п'яти» праворуч. Поставте попередній діагноз: A 25-year-old man arrived 2 hours after a traffic accident with complaints of constant intense pain in the right inguinal region and in the pelvic symphysis, inability to move independently. During examination found: the patient's position is forced - the 'frog' position (positive Volkovych's symptom). Bruises and bruises on the skin in the projection of the symphysis and the horizontal branch of the pubic bone on the right, pronounced swelling of soft tissues. Positive symptom of 'sticky heel' on the right. Put previous diagnosis:

Травматична пахвинна грижа Traumatic Groin Care

Закритий перелом лобкової кістки праворуч Closed fracture of pubic bone on the right

Вивих правого стегна Dislocation of right hip

Забій кісток тазу Pelvic fracture

Травматична аневризма стегнової артерії Traumatic femoral artery aneurysm

135 / 150
Жінка 35 років поступила зі скаргами на болі внизу живота, які посилюються під час менструації і статевих актів та іррадіюють в піхву, мажучі кров'янисті виділення до та після менструації протягом 5 днів. Безпліддя протягом 9 років. Протизапальне лікування ефекту не дало. Під час бімануального дослідження виявлено: матка збільшена, щільна, болюча, гладка. Під час гістероскопії в ділянці дна матки видно темно-червоні утвори, з яких виділяється темна кров. Який діагноз відповідає цій клінічній картині? A 35-year-old woman came in with complaints of pain in the lower abdomen, which worsens during menstruation and sexual acts and radiates into the vagina, smearing bloody discharge before and after menstruation for 5 days. Infertility for 9 years. Anti-inflammatory treatment had no effect. During a bimanual examination, it was found: the uterus is enlarged, dense, painful, smooth. During hysteroscopy, dark red formations are visible in the area of the bottom of the uterus, from which dark blood is released. What does the diagnosis correspond to this clinical picture?

Внутрішній ендометріоз Internal endometriosis

Рак матки Uterine Cancer

Ендометрит Endometritis

Гіперплазія ендометрію Endometrial hyperplasia

Міома матки Uterine myoma

136 / 150
Робітниця 40 років працює на фабриці з виробництва льону, стаж 12 років. Пред'являє скарги на задишку, кашель, слабкість. Загальний стан поліпшується у вихідні дні, але знову стає гіршим у разі виходу на роботу. У легенях вислуховуються розсіяні сухі хрипи з обох сторін. У крові спостерігається незначний лейкоцитоз. Який найвірогіднішиий діагноз? A 40-year-old female worker works at a flax factory, 12 years of experience. She complains of shortness of breath, cough, weakness. Her general condition improves on weekends, but again becomes worse when going to work. Scattered dry rales are heard in the lungs on both sides. There is a slight leukocytosis in the blood. What is the most likely diagnosis?

Силікоз Silicosis

Бронхіальна астма Bronchial asthma

Пильовий бронхіт Dust bronchitis

Алергійний альвеоліт Allergic alveolitis

Бісиноз Бісиноз

137 / 150
У 38-річної жінки після сварки виникло головокружіння, слабкість. В анамнезі вказані: анорексія, втрата ваги, нудота, діарея. Гіпотензія 50/30 мм рт.ст. у вертикальному положенні. Пульс 110 в хв, малий, ритмічний. Глюкоза крові – 3,3 мМ/л. Гіпонатріємія. Гіперкаліємія. Гіперпігментація шкіри. Екскреція 17-КС та 17-ОКС з сечею знижені. Який попередній діагноз? A 38-year-old woman developed dizziness and weakness after a quarrel. The anamnesis indicated: anorexia, weight loss, nausea, diarrhea. Hypotension 50/30 mm Hg . in an upright position. Pulse 110 per minute, small, rhythmic. Blood glucose - 3.3 mm/l. Hyponatremia. Hyperkalemia. Hyperpigmentation of the skin. Excretion of 17-KS and 17-OKS with urine is reduced. What is the previous diagnosis?

Наднирникова недостатність Adrenal insufficiency

Прихована внутрішня кровотеча Hidden internal bleeding

Вагітність, гіпотонічний стан Pregnancy, hypotonic state

Цукровий діабет, гіпоглікемічний стан Diabetes, hypoglycemic state

138 / 150
У післяопераційного хворого, якому 6 днів тому проведено видалення нижньої частки лівої легені через бактеріальну деструкцію і розвиток пневмотораксу, наросла задишка до 48/хв., РаО2 знизився до 50 мм рт. ст., РаСО2 підвищився до 65 мм рт. ст., з'явився кашель, виділення великої кількості харкотиння, загальний ціаноз шкіри, гіпотонія - 60/20 мм рт. ст. Під час аускультації правої і лівої легень прослуховується велика кількість різнокаліберних хрипів, різко ослаблене дихання. Яку невідкладну допомогу потрібно надати хворому, щоб ліквідувати дихальну недостатність? In a postoperative patient who had the lower lobe of the left lung removed 6 days ago due to bacterial destruction and the development of pneumothorax, shortness of breath increased to 48/min., PaO2 decreased to 50 mm Hg, RaCO2 increased to 65 mm Hg, a cough appeared, a large amount of sputum was released, general cyanosis of the skin, hypotension - 60/20 mm Hg. During auscultation of the right and left lungs, a large number of wheezing of various calibers, sharply weakened breathing. What emergency aid should be provided to the patient to eliminate respiratory failure?

Гіпербарична оксигенація через маску з резервуаром Hyperbaric oxygenation through a mask with a reservoir

Провести лаваж трахеобронхіального дерева Perform tracheobronchial tree lavage

Положення ортопное Position orthopnea

Пункція плевральної порожнини Puncture of the pleural cavity

Провести інтубацію трахеї з ШВЛ Carry out tracheal intubation with ventilator

139 / 150
Чоловік госпіталізований після тупої травми грудної клітки. AT -100/60 мм рт. ст., пульс - 98/хв. На рентгенограмі виявлено: ОГК без особливостей. Ехокардіографія встановила: вільна рідина в порожнині перикарду в кількості 100 мл. Через годину спостерігається зниження AT до 85/50 мм рт. ст., набухання шийних вен, пульс - 106/хв., тони серця різко ослаблені. Які найперші заходи слід вжити? A man was hospitalized after a blunt chest injury. AT - 100/60 mm Hg, pulse - 98/min. X-ray showed: OGK without features. Echocardiography established: free fluid in the pericardial cavity in the amount of 100 ml. After an hour, a decrease in AT to 85/50 mm Hg is observed, swelling of the neck veins, pulse - 106/min, heart sounds are sharply weakened. What are the first measures to be taken?

Ввести діуретики в/в Enter intravenous diuretics

Інфузія симпатоміметиків Infusion of sympathomimetics

Пункція перикарду Pericard puncture

Антибіотикотерапія Antibiotic therapy

В/в введення серцевих глікозидів IV administration of cardiac glycosides

140 / 150
У дівчинки з тетрадою Фалло, після психоемоційного збудження виник задухо-ціанотичний напад. У цьому стані мати з дитиною звернулася до педіатра, який проводив прийом у дитячій поліклініці. Який препарат невідкладної допомоги необхідно ввести першим? A girl with tetrad of Fallot, after psycho-emotional excitement, had a suffocating-cyanotic attack. In this state, the mother and the child turned to the pediatrician, who held an appointment at the children's clinic. What should an emergency drug be administered first?

Калію хлорид Potassium Chloride

Провести інтубацію трахеї і дати кисень Intubate the trachea and give oxygen

Пропранолол Propranolol

Папаверін Papaverin

Дигоксін Digoxin

141 / 150
Чоловік 36 років скаржиться на постійний тупий біль у лівому підребер'ї після прийому жирної та копченої їжі, блювоту, яка не приносить полегшення. Калові маси блискучі, з неприємним запахом. Хворіє протягом 8 років; зловживає алкоголем, багато курить. Об'єктивно встановлено: астенічної тілобудови, шкіра бліда і суха, язик з білим налітом, живіт помірно здутий. Під час пальпації - біль у зонах Шоффара, Губергриця-Скульського, точках Дежардена, Мейо-Робсона. Про яке захворювання можна подумати насамперед? A 36-year-old man complains of constant dull pain in the left hypochondrium after eating fatty and smoked food, vomiting that does not bring relief. The stools are shiny, with unpleasant smell. He has been ill for 8 years; abuses alcohol, smokes a lot. Objectively established: asthenic physique, pale and dry skin, tongue with a white coating, abdomen moderately distended. During palpation - pain in the zones of Shofar, Gubergryts-Skulsky, Desjardins points , Mayo-Robson. What disease can you think of first?

Рак голівки підшлункової залози Cancer of the head of the pancreas

Хронічний холецистит у стадії загострення Chronic cholecystitis in the acute stage

Хронічний панкреатит у стадії загострення Chronic pancreatitis in the acute stage

Виразкова хвороба у стадії загострення Ulcer disease in the acute stage

Хронічний гастро дуоденіт у стадії загострення Chronic gastroduodenitis in the acute stage

142 / 150
Жінка 50 років госпіталізована до стаціонару на 5-й день хвороби зі скаргами на сильний головний біль, температуру тіла 39 °С, безсоння. Об'єктивно встановлено: стан тяжкий, трохи збуджена, шкіра обличчя гіперемована, склери ін'єктовані. Позитивний симптом Говорова-Годельє. На тулубі та кінцівках рясний розеольозно-петехіальний висип; тахікардія, AT -140/70 мм рт. ст. Для невідкладної хіміопрофілактики у вогнищі цієї інфекції призначають: A 50-year-old woman was hospitalized on the 5th day of illness with complaints of severe headache, body temperature of 39 °C, insomnia. Objectively established: condition severe, slightly excited, the skin of the face is hyperemic, the sclera is injected. Positive Govorov-Godelier symptom. On the trunk and limbs, a profuse roseolous-petechial rash; tachycardia, AT -140/70 mm Hg. For urgent chemoprophylaxis in the focus of this infection, prescribe :

Левоміцетин Levomycetin

Канаміцин Kanamycin

Норфлоксацин Norfloxacin

Аугментин Augmentin

Доксициклін Doxycycline

143 / 150
У жінки цукровий діабет 1-го типу середнього ступеня тяжкості. Захворювання ускладнилося ретинопатією та полінейропатією. Крім того, під час неодноразового дослідження добової сечі на екскрецію альбуміна виявлена мікроальбумінурія (200-300 мг/добу). Швидкість клубочкової фільтрації -105 мл/хв. AT в межах норми. Для вторинної профілактики діабетичної нефропатії потрібно насамперед прагнути до нормалізації такого показника: The woman has type 1 diabetes mellitus of moderate severity. The disease was complicated by retinopathy and polyneuropathy. In addition, microalbuminuria was detected during repeated examination of daily urine for albumin excretion ( 200–300 mg/day).

Глікемії натщесерце Fasting blood glucose

Глікозильованного гемоглобіну Glycosylated hemoglobin

Глікемії за 2 години після їжі Glucemia 2 hours after eating

С-пептида C-peptide

Інсуліну крові Blood insulin

144 / 150
Дівчина 23 років звернулася до клініки зі скаргами на серцебиття, пітливість, слабкість, зниження працездатності, тремтіння пальців рук. Протягом місяця схудла на 10 кг. Під час огляду встановлено: частота серцевих скорочень - 120/хв., артеріальний тиск - 130/70 мм рт. ст. Під час ультразвукового дослідження (УЗД) в щитоподібній залозі виявлено вузол 2x2 см. Під час сканування в місці вузла виявлено ділянку підвищеного накопичення радіофармпрепарату. Решта тканин щитоподібної залози РФП не накопичує. Який найбільш імовірний діагноз? A 23-year-old girl came to the clinic with complaints of palpitations, sweating, weakness, reduced work capacity, tremors of the fingers. She lost 10 kg during the month. During the examination, it was found : heart rate - 120/min., blood pressure - 130/70 mm Hg. During an ultrasound examination (USG), a 2x2 cm nodule was found in the thyroid gland. During the scan, an area of increased accumulation of the radiopharmaceutical was found at the site of the nodule. The rest of the tissues RFP does not accumulate in the thyroid gland. What is the most likely diagnosis?

Змішаний токсичний зоб Mixed toxic goiter

Аутоімунний тиреоїдит Autoimmune thyroiditis

Підгострий тиреоїдит Subacute thyroiditis

Тиреотропінома Thyrotropinoma

Токсична аденома щитоподібної залози Toxic thyroid adenoma

145 / 150
Під час вивчення середнього рівня та характеру різноманітності деяких лабораторних показників отримано такі дані: для загального білку крові - середнє квадратичне відхилення ±4 г/л, коефіцієнт варіації - 6%; для швидкості осідання еритроцитів відповідно ±2 мм/год, 23%. Яка з ознак, що вивчаються, є найбільш різноманітною? During the study of the average level and nature of the diversity of some laboratory indicators, the following data were obtained: for total blood protein - mean square deviation ±4 g/l, coefficient of variation - 6 %; for the sedimentation rate of erythrocytes, respectively, ±2 mm/h, 23%. Which of the characteristics being studied is the most diverse?

Для вивчення різноманітності потрібні додаткові розрахунки The study of diversity requires additional calculations

Загальний білок сироватки крові Total serum protein

Швидкість осідання еритроцитів (ШОЕ) Erythrocyte Sedimentation Rate (ESR)

Для вивчення різноманітності потрібні додаткові дослідження More research is needed to study diversity

Відмінності в різноманітності ознак відсутні There are no differences in the variety of signs

146 / 150
Чоловік 62 років скаржиться на слабкість, жар у язиці, заніміння нижніх кінцівок. Об'єктивно встановлено: шкіра бліда з субіктеричним відтінком, язик яскраво-червоний, смакові сосочки згладжені. Печінка - +3 см. Під час гастроскопії виявлена атрофія слизової оболонки. У крові виявлено: еритроцити -1,2х10Л12/л, НЬ - 56 г/л, КП -1,4, макроцитоз, лейкоцити - 2,8х10л9/л, еозинофіли -2%, паличкоядерні -4%, сегментоядерні - 50%, лімфоцити -42%, моноцити -2%, ретикулоцити - 0,1%, тромбоцити - 120х10д9/л, ШОЕ -36 мм/год, білірубін - 29 ммоль/л, непрямий - 26 ммоль/л. Який найбільш імовірний діагноз? A 62-year-old man complains of weakness, heat in the tongue, numbness of the lower extremities. Objectively established: the skin is pale with a subicteric shade, the tongue is bright red, taste buds flattened. Liver - +3 cm. During gastroscopy, mucosal atrophy was detected. Blood revealed: erythrocytes -1.2x10L12/l, Hb - 56 g/l, CP -1.4, macrocytosis, leukocytes - 2.8x10l9/l . /l, indirect - 26 mmol/l. What is the most likely diagnosis?

Фолієво-дефіцитна анемія Folate deficiency anemia

Гемолітична анемія Hemolytic anemia

Гіпопластична анемія Hypoplastic anemia

Залізодефіцитна анемія Iron deficiency anemia

В12-дефіцитна анемія B12 deficiency anemia

147 / 150
Пацієнтка 49 років скаржиться на нерегулярність циклу впродовж 18 місяців, міжмснструальні кровотечі та «приливи», які її дуже турбують; наполягає на негайному проведенні лікування. Яку процедуру треба провести перед призначенням терапії? A 49-year-old patient complains of cycle irregularity for 18 months, intermenstrual bleeding and 'hot flashes', which bother her a lot; she insists on immediate treatment. What procedure should be performed before prescribing therapy?

Призначення медроксипрогестерону ацетату Medroxyprogesterone acetate prescription

Застосування естрогенового крему Estrogen cream application

Послідовна терапія естрогенами та прогестином Sequential estrogen and progestin therapy

Гістеректомія Hysterectomy

Біопсія ендометрію Endometrial biopsy

148 / 150
У жінки З0 років припинилась менструація, а потім почала знижуватися гострота зору. Була виявлена первинна атрофія зорових нервів, бітемпоральна геміанопсія. Упродовж 2 років хвора лікувалась окулістами амбулаторно та стаціонарно, але зір прогресивно падав. Консультація офтальмолога: гострота зору OD=0,02, OS=0,03, виражена первинна атрофія зорових нервів. Який допоміжний метод обстеження може допомогти у постановці діагнозу? A 30-year-old woman stopped menstruating, and then her visual acuity began to decrease. Primary atrophy of the optic nerves, bitemporal hemianopsia was detected. For 2 years, the patient was treated by ophthalmologists on an outpatient and inpatient basis , but vision was progressively declining. Ophthalmologist's consultation: visual acuity OD=0.02, OS=0.03, pronounced primary optic nerve atrophy. What auxiliary method of examination can help in making a diagnosis?

Дослідження ліквору Liquor Research

Електроенцефалографія Electroencephalography

Пневмоенцефалографія Pneumoencephalography

МРТ головного мозку MRI brain

Ехоенцефалографія Echoencephalography

149 / 150
Під час огляду студента лікарем було виявлено сухість та зроговіння шкіри па ліктях та колінах. На шкірі сідниць, стегон та литок зроговіння епітелію, якщо провести рукою, відчувається шорсткуватість шкіри цих ділянок, яке нагадувало «гусячу шкіру». Студент також скаржився на погіршення зору у сутінках. Яке із наведених захворювань можна припустити? During the examination of the student, the doctor found dryness and keratinization of the skin on the elbows and knees. On the skin of the buttocks, thighs, and calves, keratinization of the epithelium, if you run your hand over it, you can feel the roughness of the skin of these areas, which resembled 'goosebumps'. The student also complained of worsening vision at dusk. Which of the following diseases can be assumed?

Гіповітаміноз D Hypovitaminosis D

Гіповітаміноз А Hypovitaminosis A

Гіповітаміноз К Hypovitaminosis K

Гіповітаміноз С Hypovitaminosis C

Гіповітаміноз В6 Hypovitaminosis B6

150 / 150
У чоловіка на різних ділянках шкірного покриву, переважно симетрично, розташовані депігментовані плями, що не лущаться, різного розміру й обрисів. Частина волосся на них знебарвлена. Захворювання почалося в дитинстві. Появі плям не передували будь-які елементи висипки. Який імовірний діагноз? The man has depigmented spots that do not peel, of different sizes and outlines, on different areas of the skin, mostly symmetrically. Some of the hair on them is discolored. The disease began in childhood. The appearance of spots was not preceded by any elements of the rash. What is the probable diagnosis?

Рожевий лишай Pink lichen

Вітиліго Vitiligo

Псоріаз Psoriasis

Альбінізм Albinism

Різнокольоровий лишай Multicolored lichen